mcat full length5

81
Physical Sciences Time: 100 Minutes Questions 1–77 DO NO T BEGIN THIS SECTION UNTIL Y OU ARE T OLD T O DO SO . Full-Length Test 5 06/27/2003 12:16 PM Page 1

Upload: ali

Post on 28-Apr-2015

236 views

Category:

Documents


20 download

TRANSCRIPT

Page 1: MCAT Full Length5

Physical SciencesTime: 100 Minutes

Questions 1–77

DO NOT BEGIN THIS SECTION UNTIL YOU ARE TOLD TO DO SO.

Full-Length Test 5 06/27/2003 12:16 PM Page 1

Page 2: MCAT Full Length5

2

GO ON TO THE NEXT PAGE.

PHYSICAL SCIENCES

DIRECTIONS: Most of the questions in the PhysicalSciences test are organized into groups, with adescriptive passage preceding each group of ques-tions. Study the passage, then select the single bestanswer to each question in the group. Some of thequestions are not based on a descriptive passage; youmust also select the best answer to these questions. Ifyou are unsure of the best answer, eliminate thechoices that you know are incorrect, then select ananswer from the choices that remain. Indicate yourselection by blackening the corresponding circle onyour answer sheet. A periodic table is provided belowfor your use with the questions.

1

H

1.0

2

He

4.0

3

Li

6.9

4

Be

9.0

5

B

10.8

6

C

12.0

7

N

14.0

8

O

16.0

9

F

19.0

10

Ne

20.2

11

Na

23.0

12

Mg

24.3

13

Al

27.0

14

Si

28.1

15

P

31.0

16

S

32.1

17

Cl

35.5

18

Ar

39.9

19

K

39.1

20

Ca

40.1

21

Sc

45.0

22

Ti

47.9

23

V

50.9

24

Cr

52.0

25

Mn

54.9

26

Fe

55.8

27

Co

58.9

28

Ni

58.7

29

Cu

63.5

30

Zn

65.4

31

Ga

69.7

32

Ge

72.6

33

As

74.9

34

Se

79.0

35

Br

79.9

36

Kr

83.8

37

Rb

85.5

38

Sr

87.6

39

Y

88.9

40

Zr

91.2

41

Nb

92.9

42

Mo

95.9

43

Tc

(98)

44

Ru

101.1

45

Rh

102.9

46

Pd

106.4

47

Ag

107.9

48

Cd

112.4

49

In

114.8

50

Sn

118.7

51

Sb

121.8

52

Te

127.6

53

I

126.9

54

Xe

131.3

55

Cs

132.9

56

Ba

137.3

57

La *

138.9

72

Hf

178.5

73

Ta

180.9

74

W

183.9

75

Re

186.2

76

Os

190.2

77

Ir

192.2

78

Pt

195.1

79

Au

197.0

80

Hg

200.6

81

Tl

204.4

82

Pb

207.2

83

Bi

209.0

84

Po

(209)

85

At

(210)

86

Rn

(222)

87

Fr

(223)

88

Ra

226.0

89

Ac †

227.0

104

Rf

(261)

105

Ha

(262)

106

Unh

(263)

107

Uns

(262)

108

Uno

(265)

109

Une

(267)

*

58

Ce

140.1

59

Pr

140.9

60

Nd

144.2

61

Pm

(145)

62

Sm

150.4

63

Eu

152.0

64

Gd

157.3

65

Tb

158.9

66

Dy

162.5

67

Ho

164.9

68

Er

167.3

69

Tm

168.9

70

Yb

173.0

71

Lu

175.0

90

Th

232.0

91

Pa

(231)

92

U

238.0

93

Np

(237)

94

Pu

(244)

95

Am

(243)

96

Cm

(247)

97

Bk

(247)

98

Cf

(251)

99

Es

(252)

100

Fm

(257)

101

Md

(258)

102

No

(259)

103

Lr

(260)

PERIODIC TABLE OF THE ELEMENTS

Full-Length Test 5 06/27/2003 12:16 PM Page 2

Page 3: MCAT Full Length5

Passage I (Questions 1–5)

An object totally immersed in water is subject to twoopposing forces which act vertically. First, the object’sweight, which is a result of the Earth’s gravitational attrac-tion, pulls it downward. Second, there is a buoyant forceexerted by the water on the object, which pushes the objectupward. Archimedes’ principle states that the upwardbuoyant force exerted by the water on the object is equalin magnitude to the weight of the water displaced.

A researcher constructs an apparatus, shown in Figure 1,to investigate the forces on inflatable devices used byscuba divers. A heavy, elastic balloon filled with an idealgas is immersed in a pool of water. At depth d1, the balloonis in equilibrium. In other words, the weight of the balloonis exactly equal to the buoyant force exerted by the wateron the balloon. Because the net force is zero, the balloon issuspended motionless in water.

Since the balloon is elastic and is filled with an idealgas, its volume will vary with pressure and temperature.This effect is significant because the pressure at all pointsin the pool is not a constant. (Note: Assume that the tem-perature at all points in the pool is constant.)

Figure 1 Balloon of ideal gas immersed in a pool of water

1. If the volume of the balloon at depth d1 is doubledand the mass is unchanged, which of the followingwill also double?

A. The weight of the balloon

B. The density of the balloon

C. The density of the water that the balloon dis-places

D. The buoyant force on the balloon

2. If the water is replaced by a denser fluid, then whichof the following will NOT occur?

A. The pressure at depth d1 will increase.

B. The density of the balloon will remain the same.

C. The volume of the balloon will decrease.

D. The weight of the balloon will remain the same.

3. A second object besides the balloon is suspendedmotionless when released at depth d1. What can beconcluded?

A. Its mass is equal to that of the balloon.

B. Its volume is equal to that of the balloon.

C. Its density is equal to that of the balloon.

D. The buoyant force on it is equal to the buoyantforce on the balloon.

4. The balloon is moved to depth d2 and then released.Which way does the balloon go?

A. Upwards, because the weight is greater than thebuoyant force.

B. Downwards, because the buoyant force isgreater than the weight.

C. Downwards, because the weight is greater thanthe buoyant force.

D. The balloon does not move, because the weightis equal to the buoyant force.

5. An iron ball with a volume of 4 × 10–5 m3 is releasedfrom a height of 0.35 m above the bottom of a 5-meterdeep pool of water. How long does it take the ball toreach the bottom? (Note: The density of iron is 7.9 × 103 kg/m3, and the density of water is 1.00 × 103

kg/m3. Assume the viscosity of the water is negligible.)

A. 0.03 s

B. 0.3 s

C. 30 s

D. 300 s

balloon

water

d1

d2

3

GO ON TO THE NEXT PAGE.

Full-Length Test 5 06/27/2003 12:16 PM Page 3

Page 4: MCAT Full Length5

Passage II (Questions 6–12)

Although water is a popular inorganic solvent, othersolvents are often used for their particular properties. Onesuch solvent is liquid ammonia. Pure ammonia has a freez-ing point of –77°C and a boiling point of –33.35°C. Itexists as a gas at room temperature. However, sinceammonia can be liquefied under pressure, it is sold incylinders that dispense ammonia as a liquid.

Liquid ammonia, like water, undergoes self-ionization(Reaction 1).

2NH3(l) NH4+(am) + NH2

–(am) K–50°C = 10–30

Reaction 1

(Note: am = ammoniacal solution)

Even though liquid ammonia and water are both used todissolve ionic compounds, these compounds can exhibit dif-ferent behavior in each solvent. For example, ammonia is astronger base than water; therefore, weak acids that do notcompletely ionize in water may do so in liquid ammonia.

Another property of liquid ammonia that differs fromwater is its complexing ability: The silver halide salts,which are insoluble in water, are very soluble in liquidammonia (Reaction 2).

AgX(s) + xNH3(l) Ag(NH3)x+(am) + X– (am)

Reaction 2

A student prepared four solutions as follows:

Solution 1: 2 moles of CaCl2 dissolved in 1500 g ofliquid ammonia.

Solution 2: 2 moles of CaCl2 dissolved in 1500 g ofwater.

Solution 3: 2 moles of CH3COOH dissolved in 2000 gof water.

Solution 4: 2 moles of CH3COOH dissolved in 2000 gof liquid ammonia.

(Note: The freezing point depression constant, Kf, ofNH3 is 0.957°C/m; that of water is 1.86°C/m; the boilingpoint elevation constant, Kb, of water is 0.52°C/m.)

6. Which of the following is a TRUE statement?

A. The amount of undissociated acetic acid islarger in solution 4 than in solution 3.

B. The freezing-point depression of solution 3 andsolution 4 are equal.

C. The freezing-point depression of solution 3 islarger than that of solution 4.

D. The freezing-point depression of solution 3 issmaller than that of solution 4.

7. What is the effect on the freezing point and boilingpoint if 3 moles of solid silver nitrate are added tosolution 2?

A. The boiling point increases and the freezingpoint decreases.

B. Neither the boiling point nor the freezing pointchanges.

C. The boiling point decreases and the freezingpoint increases.

D. Both the boiling point and the freezing pointdecrease.

8. Ammonium chloride is added to the liquid ammoniasolution. As a result, the equilibrium of Reaction 1:

A. will shift to the left.

B. will shift to the right.

C. will not be affected.

D. will be reached faster.

4

GO ON TO THE NEXT PAGE.

Full-Length Test 5 06/27/2003 12:16 PM Page 4

Page 5: MCAT Full Length5

9. Which of the following represents the volume of gasformed (in liters) at STP when 11 cm3 of liquidammonia is completely vaporized? (Note: the densityof liquid ammonia is 0.682 g/cm3.)

A. (0.682 × 11/17) × 22.4

B. 0.682 × 11/17

C. (0.682 × 22.4)/(11 × 17)

D. (0.682 × 17)/(22.4 × 11)

10. The student prepared another solution containing1500 g of water and an unknown amount of calciumchloride. The boiling point increased to 101.56°C.What is the mass of calcium chloride dissolved inthis sample?

A. 0.167 g

B. 1.67 g

C. 16.7 g

D. 167 g

11. What is the equilibrium constant for Reaction 2?

A. [Ag(NH3)x+][X–]

B.

C.

D.

12. Which of the following accurately describes theaction of ammonia in Reaction 2?

A. It acts as a Brønsted acid.

B. It acts as a Brønsted base.

C. It acts as a Lewis acid.

D. It acts as a Lewis base.

[Ag(NH3)x+][X–]

[NH3]x

[AgX][NH3]x

[Ag(NH3)x

+][X–]

[Ag(NH3)x+]

[NH3]x

5

GO ON TO THE NEXT PAGE.

Full-Length Test 5 06/27/2003 12:16 PM Page 5

Page 6: MCAT Full Length5

Passage III (Questions 13–17)

The magnitude of the magnetic field at the center of acircular loop is given by B = µ0i/2r, where i is the currentthrough the wire, r is the radius of the loop, and µ0 is aconstant. Figure 1 shows an electric circuit, lying in theplane of the page, designed to generate a magnetic field. Avoltage source is connected in series to a switch, a 3-Ωresistor, a circular wire loop, and a 1-Ω resistor. A 2-Ωresistor is connected in parallel with the 1-Ω resistor. Theswitch, shown in the open position, prevents the flow ofcurrent when it is in the open position. Two experimentsare conducted with the apparatus.

Experiment 1

The voltage source is set to provide 10 V. At time t = 0, the switch is closed. The magnitude and direction ofthe magnetic field generated by the circular loop are mea-sured at points P and Q.

Experiment 2

The switch is closed at time t = 0, and the voltagesource provides a voltage which varies in time. Figure 2shows the graph of this voltage. The magnitude of themagnetic field at point P is measured as a function of time.

Figure 1 Electric circuit designed to generate a magneticfield

Figure 2 Output voltage of voltage source as a function oftime

13. In Experiment 1, which of the following woulddecrease the magnetic field at point P?

I. Increasing R1II. Increasing R2

III. Increasing R3IV. Decreasing r

A. I only

B. I and IV only

C. I, II, and III only

D. I, II, III and IV

14. In Experiment 1, after the switch is closed, what isthe orientation of the magnetic field at P and Q,respectively?

A. Into the page, out of the page

B. Out of the page, into the page

C. Into the page, into the page

D. To the left, to the right

15. If the current through R1 is 2.7 A, what is the currentthrough R2?

A. 0.9 A

B. 1.8 A

C. 2.7 A

D. 8.1 A

Time

Vol

tage

switch

R1 = 3Ω

R2 = 3Ω

R3 = 3Ω

rP Q

voltagesource

+–

6

GO ON TO THE NEXT PAGE.

Full-Length Test 5 06/27/2003 12:16 PM Page 6

Page 7: MCAT Full Length5

16. In Experiment 2, which of the following graphs rep-resents the magnitude of the magnetic field at point Pas a function of time?

A. C.

B. D.

17. The instantaneous power dissipated by the resistorsin Experiment 2 is shown below as a function oftime. What quantity is represented by the area underthe curve?

A. Energy

B. Current

C. Resistance

D. Voltage

Time

Pow

er

7

GO ON TO THE NEXT PAGE.

Time

B

Time

B

Time

B

Time

B

Full-Length Test 5 06/27/2003 12:16 PM Page 7

Page 8: MCAT Full Length5

Passage IV (Questions 18–23)

One of the most important uses of lead today is in theproduction of automobile batteries. These batteries consistof six lead-acid cells connected in series, and each cellgenerates approximately 2V during discharge. The lead-acid cell is an example of a secondary cell—an electro-chemical cell that has to be charged by some otherelectrical source before it can be used. A secondary cell,therefore, acts as an electrolytic cell when charged and agalvanic cell during discharge.

Lead-acid cells contain a number of lead (II) sulfatecovered grids or electrodes immersed in an aqueous solu-tion of sulfuric acid. These grids have a large surface areaso that a large current can be generated rather quickly, i.e.,when the cells discharge and the car engine is started.

When the cell is charged, lead (II) sulfate undergoesthe following two reactions: it is reduced to form lead atthe cathode, and it is oxidized to form lead (IV) oxide atthe anode. When the cell discharges, the reverse reactionsoccur; that is, lead is oxidized at the anode to form lead (II)sulfate (Reaction 1) and lead (IV) oxide is reduced at thecathode to form lead (II) sulfate (Reaction 2).

Pb(s) + SO42–(aq) → PbSO4(s) + 2e– Eo = +0.36 V

Reaction 1

PbO2(s) + SO42–(aq) + 4H+(aq) + 2e– → PbSO4(s) +

2H2O(l)

Eo = +1.69V

Reaction 2

The lead-acid cell should only be charged up to thepoint where all the lead (II) sulfate is consumed; beyondthis point, water can undergo electrolysis to generategaseous oxygen and hydrogen, both of which can react withthe newly formed deposits of lead and lead (IV) oxide.

Conversely, the cell should not be stored for long peri-ods of time, or allowed to discharge to the point wherethere is more than a 30% conversion of lead and lead (IV)oxide since lead sulfate can obstruct the flow of sulfuricacid and damage the lead grids.

18. When a car engine was started for 3 seconds, a cur-rent of 333 amps was generated by the battery. Howmany grams of lead are used up during this time?

[Note: Faraday’s constant = 9.6485 104 C/mol]

A. 0.0104 g

B. 0.104 g

C. 1.04 g

D. 10.4 g

19. What would be the effect on the discharge voltage ifa small amount of lead sulfate was added to a cellimmediately after it had been charged?

A. The voltage would not change.

B. The voltage would decrease.

C. The voltage would increase.

D. The voltage could not be measured.

20. After six months, the output voltage of a car batterydropped from 12V to 6V. What is the most likelyexplanation for this observation?

A. There is less than 30% conversion of lead andlead (IV) oxide to lead sulfate.

B. Half of the lead-acid cells give no output volt-age.

C. The level of aqueous sulfuric acid in the cells istoo low.

D. There is 100% conversion of lead and lead (IV)oxide to lead (II) sulfate in all the cells.

8

GO ON TO THE NEXT PAGE.

Full-Length Test 5 06/27/2003 12:16 PM Page 8

Page 9: MCAT Full Length5

21. Which of the following represents the anode half-reaction when the lead-acid battery is overcharged?

A. 2H2O(l) → O2(g) + 4H+(aq) + 4e–

B. 4H+(aq) + 4e– → 2H2(g)

C. 2H2O(l) → 2H2(g) + O2(g)

D. 2H2O(l) + 2e– → H2(g) + 2OH–(aq)

22. Which of the following procedures would be themost useful in preventing the loss of capacity of abattery while in storage?

A. The addition of lead sulfate to the electrolyte

B. Sealing the electrolyte in the battery to preventgas loss

C. The addition of sulfuric acid to the electrolyte

D. The addition of lead and lead (IV) oxide to theelectrolyte

23. What is the balanced equation for the discharge reac-tion in the lead-acid battery?

A. 2PbSO4(s) + 2H2O(l) →Pb(s) + PbO2(s) + 4H+(aq) + 2SO4

2–(aq)

B. Pb(s) + PbO2(s) + 2SO42–(aq) →

2PbSO4(s) + O2(g)

C. 2Pb(s) + 2SO42–(aq) → 2PbSO4(s)

D. Pb(s) + PbO2(s) + 4H+(aq) + 2SO42–(aq) →

2PbSO4(s) + 2H2O(l)

9

GO ON TO THE NEXT PAGE.

Full-Length Test 5 06/27/2003 12:16 PM Page 9

Page 10: MCAT Full Length5

Questions 24 through 28 are NOT based ona descriptive passage.

24. A steel flagpole is found to be 20 meters high on aday when the temperature is 25°C. How would thelength of the flagpole change if the temperature dropsto –5°C? (Note: The coefficient of linear expansionfor steel is 11 10–6 K–1.)

A. It would increase by 6.6 mm.

B. It would increase by 0.33 mm.

C. It would decrease by 0.33 mm.

D. It would decrease by 6.6 mm.

25. What is the oxidation state of chlorine in ClO4–?

A. –1

B. 0

C. +7

D. +9

26 How many moles of Ba2+ are found in a 500 mLsolution of barium sulfate? (Note: The Ksp of BaSO4 = 1.1 10–10.)

A. 1 × 10–6

B. 5 × 10–6

C. 1 × 10–5

D. 5 × 10–5

27. Aluminum oxide readily undergoes the followingreactions:

Al(OH)3(s) + 3H+(aq) → Al3+(aq) + 3H2O(l)

Al(OH)3(s) + OH–(aq) → Al(OH)4–(aq)

Based on these reactions, aluminum oxide can beclassed as:

A. an amphoteric compound.

B. an acidic compound.

C. a basic compound.

D. a neutral compound.

28. If a flask containing 0.28 g of nitrogen and 0.64 g ofoxygen has a total pressure of 3 atm, what is the par-tial pressure of nitrogen in the flask?

A. 0.5 atm

B. 1.0 atm

C. 2.0 atm

D. 3.0 atm

10

GO ON TO THE NEXT PAGE.

Full-Length Test 5 06/27/2003 12:16 PM Page 10

Page 11: MCAT Full Length5

Passage V (Questions 29–33)

The collision theory states that molecules must collidewith each other before they can react; the rate at whichthey collide and react determines the rate of a reaction. Ifthe reactant molecules possess enough energy, they willcombine to form an activated complex—a very short-livedspecies—which dissociates into product molecules.

Consistent with the collision theory, two differentmechanisms have been proposed to account for the reac-tion of iodine with hydrogen. In the first mechanism,molecular hydrogen and molecular iodine collide to forman activated complex. The activated complex then dissoci-ates into two molecules of hydrogen iodide (Figure 1).

Figure 1 First proposed mechanism for the formation ofhydrogen iodide

Since the formation of the activated complex resultsfrom the collision between two molecules (hydrogen andiodine), the reaction is classed as bimolecular. The overallrate of the reaction depends on the concentration of hydro-gen and iodine:

H2 (g) + I2 (g) 2HI(g)

rate of reaction = k[H2][I2]

Mechanism 1

However, recent experimental investigation suggeststhat the reaction mechanism is more complex. A secondmechanism has been proposed: Initially, iodine dissociatesinto two iodine atoms; these atoms then react with molec-ular hydrogen to form two molecules of hydrogen iodide(Figure 2).

Figure 2 Second proposed mechanism for the formation ofhydrogen iodide

In this case, the formation of the activated complexresults from the collision of three species, so the reactionis classed as termolecular. The proposed kinetics for thisprocess are as follows:

I2(g) 2I(g) (fast)

2I(g) + H2(g) 2HI(g) (slow)

rate of reaction = k3[I]2[H2]

Mechanism 2

In the termolecular reaction, the overall rate of reactionis dependent on the concentration of molecular hydrogenand atomic iodine—an unstable intermediate. Since theconcentration of atomic iodine cannot be measured, wemake the approximation (known as the steady-stateapproximation) that the rate of production and consump-tion of atomic iodine is the same, so its concentration doesnot change. Therefore:

k1[I2] = k2[I]2

and rearranging this equation gives:

[I]2 = [I2]

Equation 1

Since the concentration of molecular iodine can bemeasured, substituting Equation 1 into the reaction equa-tion in mechanism 2 gives:

rate of reaction = [I2][H2]

Equation 2

The second mechanism, therefore, is similar to the firstin that the rate of reaction is dependent on the concentra-tion of iodine and hydrogen.

To investigate these two mechanisms, a chemistobtained the following data:

Initial H2 Initial I2 Initial rate ofconcentration concentration formation of

(mol/L) (mol/L) HI (mol/L•s)

0.25 0.25 7.5 10–6

0.25 0.50 1.5 10–5

0.50 0.50 3.0 10–5

Table 1

k1k3k2

k1k2

+

I2 2I H2 2HIActivatedComplex

+

I2 H2 2HIActivatedComplex

11

GO ON TO THE NEXT PAGE.

k

k1

k3

k2

Full-Length Test 5 06/27/2003 12:16 PM Page 11

Page 12: MCAT Full Length5

29. Which of the following statements is true?

A. The data obtained by the chemist supportsmechanism 1, but not mechanism 2.

B. The data obtained by the chemist supportsmechanism 2, but not mechanism 1.

C. The data obtained by the chemist supportsmechanism 1 and mechanism 2.

D. The data obtained by the chemist supports nei-ther mechanism 1 nor mechanism 2.

30. The change in enthalpy of the slow step in mecha-nism 2 is 33 kcal/mol K, and the change in entropy is2 kcal/mol. Is this step spontaneous?

A. No, because ∆G is positive.

B. No, because hydrogen iodide is an unstablemolecule.

C. Yes, because ∆G is negative.

D. Cannot be determined without more information

31. Assuming mechanism 1 is correct, what is the rateconstant, k, for the reaction?

A. 7.5 10–6 L/mol•s

B. 6.0 10–5 L/mol•s

C. 1.2 10–4 L/mol•s

D. 2.4 10–4 L/mol•s

32. In mechanism 2, the reaction is second order withrespect to:

A. atomic iodine and hydrogen.

B. atomic iodine.

C. molecular iodine.

D. hydrogen.

33. What is the enthalpy of formation, ∆Hf, of hydrogeniodide at 298 K? (Note: The bond energies of H2(g),I2(g), and HI(g) at 298 K are 436 kJ/mol, 151 kJ/mol,and 298 kJ/mol, respectively.)

A. 9 kJ/mol

B. 4.5 kJ/mol

C. –4.5 kJ/mol

D. –9 kJ/mol

12

GO ON TO THE NEXT PAGE.

Full-Length Test 5 06/27/2003 12:16 PM Page 12

Page 13: MCAT Full Length5

Passage VI (Questions 34–38)

An apparatus known as Kundt’s tube is set up to mea-sure the speed of sound in aluminum. A schematic of theapparatus is shown in Figure 1. A glass tube is laid hori-zontally with cork dust spread over the bottom. One end ofthe tube is closed, while the other is attached via a washerto an aluminum rod clamped at the center. A rag coveredwith violin rosin is used to stroke the rod, setting up longi-tudinal vibrations in the aluminum rod. The glass tube isthen moved back and forth so that the length of the air col-umn can sustain standing waves. The cork dust is seen toclump at the nodes.

The dust clusters are found to be about 5.5 cm apart. Thisvalue can be used in conjunction with the speed of sound inair (obtained from literature) to determine the frequency ofthe vibrations in the air, and hence in the aluminum rod. Thespeed of sound in aluminum can then be determined usingthe fact that a node occurs in the middle of the rod (where itis clamped) and antinodes occur at the ends.

Figure 1 Kundt’s tube apparatus

34. The speed of sound in an ideal, diatomic gas is givenby v = 1.40P/ρ, where P is the pressure and ρ is thedensity. In which of the following gases is the speedof sound the fastest?

A. Hydrogen at 20°C

B. Hydrogen at 0°C

C. Air at 20°C

D. Air at 0°C

35. What is the frequency of the vibrations in the rod?(Speed of sound in air = 344 m/s)

A. Cannot be determined without the length of therod

B. Cannot be determined without the length of thetube

C. 6200 Hz

D. 3100 Hz

36. What is the wavelength of the fundamental vibrationin the aluminum rod, if the length of the rod is L?

A. 4 L

B. L

C. 1/2 L

D. 2 L

37. The standing wave sustained in the glass tube is:

A. mechanical and transverse.

B. mechanical and longitudinal.

C. electromagnetic and transverse.

D. electromagnetic and longitudinal.

38. Which of the following should be true of the Kundt’stube setup?

A. The tube should be evacuated.

B. The tube should be sealed with air at an initialpressure of 1 atm; no air must be exchangedwith the environment.

C. The tube should be sealed with air at an initialdensity equal to that of aluminum; no air mustbe exchanged with the environment.

D. The tube should allow air to enter and exit freelysomewhere.

clamp

aluminumrod

cork dust collects at nodes

13

GO ON TO THE NEXT PAGE.

Full-Length Test 5 06/27/2003 12:16 PM Page 13

Page 14: MCAT Full Length5

Passage VII (Questions 39–44)

Short wavelength electromagnetic rays called X-rays areoften used to image internal anatomical structures. Someparts of the body are nearly transparent to X-rays, while oth-ers are relatively opaque. A picture is taken of the “shadow”cast under X-ray illumination. The intensity of a beam of X-rays transmitted through a medium is given by I = Ioe–mx,where Io is the intensity incident on the medium, µ is theabsorption coefficient of the medium, and x is the distancetraveled through the medium. Like other electromagneticwaves, the frequency f and wavelength λ of a beam of X-rayssatisfy the relations c = λf and E = hf, where c is the speed oflight in a vacuum, E is the energy of a photon, and h isPlanck’s constant. (Note: hc = 1240 nm•eV.)

X-rays interact with atoms primarily via collisionswith electrons in low atomic energy levels. The probabil-ity of a collision with an electron in a neutral atom is pro-portional to the fourth power of the nuclear charge. In thistype of collision, the electron is frequently ejected from theatom. A higher level electron “falls down” to fill thevacancy, emitting a long wavelength photon in the processand creating a new vacancy at a higher energy level. Ahigher level electron fills the new vacancy. This processcontinues until there are no electrons at higher energy lev-els to fall to a lower state, leaving a single vacancy. Theatom is then singly-ionized. This rapid emission of pho-tons is called a cascade.

Geiger counters measure the intensity of X-rays bymaintaining a high voltage difference between a wire andthe metal wall of a gas chamber. Free electrons created byradiation-induced ionization are accelerated toward thewire so rapidly they ionize even more atoms in the gas.This ionization avalanche causes the counter to “click.”

39. Tin has ten naturally occurring non-radioactive iso-topes, with atomic masses of 112, 114, 115, 116, 117,118, 119, 120, 122, 124. Which of these would youexpect to scatter the most X-ray radiation?

A. The isotope with atomic mass 112

B. The isotope with atomic mass 124

C. A mixture of isotopes in the natural abundances

D. All would scatter X-rays equally well

40. X-rays are typically designated as those electromag-netic waves between about 0.01 nm and 1 nm. Whichof the following energies might an X-ray photonhave?

A. 1 MeV

B. 10 keV

C. 100 eV

D. 10 eV

41. The X-rays that doctors actually look at are, in fact,negatives of the shadow. Bones and connective tissueare usually light, while soft tissue is dark. This isbecause:

A. the greater relative concentration of heavy ele-ments found in bones and connective tissuescatters more of the incident X-rays.

B. heavy elements are more common in bones andconnective tissue resulting in greater emissionof the lower energy cascade photons.

C. heavy elements are less common in bones andconnective tissue allowing more incident X-raysto show through.

D. the light elements in soft tissue absorb the cas-cade photons, increasing the relative brightnessof the unscattered X-rays.

42. Compared to the incident X-rays, the cascade pho-tons have:

A. lower energy.

B. shorter wavelength.

C. higher speed.

D. higher frequency.

14

GO ON TO THE NEXT PAGE.

Full-Length Test 5 06/27/2003 12:16 PM Page 14

Page 15: MCAT Full Length5

43. What is the ratio of the intensity of an X-ray beamafter it passes through 1 cm of plasma to the intensityof the beam after it passes through 6 cm of plasma?(Note: Plasma has an absorption coefficient ofapproximately 0.2 cm–1.)

A. 1/e

B. e0.2

C. e

D. e2

44. In order for the free electrons in a Geiger counter tobe accelerated toward the wire:

A. the wall of the gas chamber must be at a lowerpotential than the wire.

B. the wall of the gas chamber must be at a higherpotential than the wire.

C. the wall of the gas chamber and the wire mustbe connected by an equipotential surface.

D. the wire must be negatively charged.

15

GO ON TO THE NEXT PAGE.

Full-Length Test 5 06/27/2003 12:16 PM Page 15

Page 16: MCAT Full Length5

Passage VIII (Questions 45–50)

Gravimetric analysis is an analytical technique basedon the measurement of weight. This technique oftenreveals information about the composition of a compound.

Thermogravimetric analysis (TGA) monitors weight asa function of temperature. The apparatus is shown in Fig-ure 1. The sample to be analyzed is placed in a cup that isencased in a gas-filled chamber. The nature of the gas canbe varied to make the atmosphere around the sample oxi-dizing, reducing, inert, etc. The furnace surrounding thechamber ensures the temperature of the sample can be var-ied while its weight is still monitored.

Figure 1 TGA Apparatus

A student decided to carry out two experiments usingTGA. In the first experiment, nickel powder (50 mg) wasplaced in the sample cup and the chamber was evacuatedand then filled with oxygen at a rate of 0.1 L/min. Thesample was then heated to 1000 °C at a rate of 10 °C/min.Upon heating, the weight of the sample increased to 64mg. The sample was then cooled to room temperature andthe chamber was evacuated. During this time, the weightof the sample did not change. However, upon filling thechamber with hydrogen and heating the sample as before,the weight of the sample dropped to 50 mg.

In a second experiment, 50 mg of calcium oxalate(CaC2O4•H2O) was placed in the sample cup. The chamberwas filled with air, and the sample was heated to 1000 °Cat a rate of 10 °C/min. The student measured the weight ofthe sample at 50 °C intervals, and obtained the followinggraph:

Figure 2 Graph of sample weight vs. temperature

45. Which of the following accurately represents thereaction between nickel and oxygen?

A. 2Ni(s) + 3/2O2(g) → Ni2O3(s)

B. Ni2+(aq) + O2–(aq) → NiO(s)

C. Ni(s) + 1/2O2(g) → NiO(s)

D. Ni(s) + O2(g) → NiO2(s)

46. How many moles of product are formed when nickelreacts with oxygen?

A. 0.86

B. 0.43

C. 8.6 10–4

D. 4.3 10–4

47. Would TGA be a suitable method to detect the occur-rence of a phase change in a compound?

A. Yes, because the phase change is accompaniedby an increase in weight.

B. Yes, because the phase change is accompaniedby a decrease in weight.

C. No, because the molecular structure of the sampledoes not change.

D. No, because the phase change is not accompa-nied by a change in weight.

0

50

500

CaC2O4–H2O

Compound A

Compound B

Compound C

1000

Temperature (°C)

Exit Gas

Inlet Gas

Microbalance

FurnaceSample Cup

16

GO ON TO THE NEXT PAGE.

Full-Length Test 5 06/27/2003 12:16 PM Page 16

Page 17: MCAT Full Length5

48. Unlike nickel(II) oxide, iron(II) oxide is easily oxi-dized to iron(III) oxide. This is because:

A. Fe3+ contains a half-filled d subshell, which isvery stable.

B. nickel cannot possess oxidation states higherthan +2.

C. Ni2+ contains a half-filled d subshell, which isvery stable.

D. iron(II) oxides are very unstable compounds.

49. What products are formed when the nickel sample istreated with hydrogen?

A. Nickel(II) oxide and water

B. Nickel and water

C. Nickel hydride and oxygen

D. Nickel and hydrogen peroxide

50. When water is added to Compound C, a very strongbase is formed. Compound C is most likely:

A. calcium oxalate.

B. calcium oxide.

C. calcium carbonate.

D. calcium hydroxide.

17

GO ON TO THE NEXT PAGE.

Full-Length Test 5 06/27/2003 12:16 PM Page 17

Page 18: MCAT Full Length5

Questions 51 through 55 are NOT based ona descriptive passage.

51. Which of the following molecules is nonpolar?

A. PCl3B. SO2C. CH3Cl

D. BF3

52. Three charges of equal magnitudes but unequal signsare placed at the corners of an isosceles triangle.Charges Q and R are equidistant from charge P andare fixed so that they cannot move. What is the initialvelocity vector of charge P as it is released?

A. A

B. B

C. C

D. D

53. A block is attached to a vertical spring and set intooscillation. Which of the following will increase theperiod of the resulting oscillations?

I. Increasing the mass of the block

II. Increasing the amplitude of the oscillation

III. Decreasing the spring constant of thespring

A. I only

B. I and II only

C. I and III only

D. II and III only

54. A helicopter flying west at 10 mph starts to increaseits altitude at a rate of 5 mph. If it passes through aregion of space in which there exists an air current of10 mph moving from north to south, how long will ittake the helicopter to travel 3 miles?

A. 7.2 minutes

B. 12 minutes

C. 18 minutes

D. 20 minutes

55. The following reaction is an example of:

Zn(s) + CuSO4(aq) → ZnSO4(aq) + Cu(s)

A. a single displacement reaction.

B. a metathesis reaction.

C. a decomposition reaction.

D. a disproportionation reaction.

+

+

A B

Q R

PC

D

18

GO ON TO THE NEXT PAGE.

Full-Length Test 5 06/27/2003 12:16 PM Page 18

Page 19: MCAT Full Length5

Passage IX (Questions 56–60)

Natural light can be polarized upon reflection asshown in Figure 1. An unpolarized light beam in air isincident on a dielectric surface (glass) at an angle θi equalto the Brewster angle. Since the light is incident at theBrewster angle, a right angle is formed between therefracted and reflected rays, and the reflected ray is lin-early polarized. The electric field vector of the reflectedray is parallel to the glass surface. The relationshipbetween the angle of incidence θi and the angle of refrac-tion θt is given by Snell’s law: nAsinθi = nGsinθt, wherenA is the index of refraction of air and nG is the index ofrefraction of glass.

Figure 2 shows an unpolarized light source illuminatingthe linear polarizers P1 and P2 at normal incidence. Unpo-larized light incident on a polarizer can be represented asthe superposition of any two waves with electric field vectorsthat are equal in magnitude and perpendicular to each other.When unpolarized light is incident on a polarizer, the polar-izer transmits 100% of the electric field component parallelto its transmission axis, but none of the perpendicular com-ponent. According to Malus’ law, the intensity of lighttransmitted through polarizer P2 is given by I1cos2φ, whereI1 is the intensity of the light transmitted through P1, and φis the angle between the transmission axes of the polarizersP1 and P2. (Note: cosθ = sin(90° – θ))

Figure 1 Unpolarized light incident on glass at the Brewsterangle

Figure 2 Unpolarized light incident on polarizers P1 and P2

56. The orientation of the transmission axis of a polarizercan be determined by looking through it into thereflected beam of Fig. 1. Which statement accuratelydescribes what is observed as the polarizer is rotated?

A. The light intensity transmitted through thepolarizer is at a minimum when its transmissionaxis is parallel to the glass surface.

B. The light intensity transmitted through thepolarizer is at a minimum when its transmissionaxis is perpendicular to the glass surface.

C. The light intensity transmitted through thepolarizer is at a minimum when its transmissionaxis is at an angle of 45° with respect to theglass surface.

D. The light intensity transmitted through the polar-izer is independent of the polarizer’s orientation.

57. Longitudinal waves, such as sound waves cannot bepolarized because:

A. they are not reflected at the boundary betweentwo media.

B. they are not attenuated when they pass from onemedium to another.

C. they have no electric field component.

D. their vibration is not perpendicular to the propa-gation direction.

P1

Light Source A B

P2

glass

rθiθ

light source

19

GO ON TO THE NEXT PAGE.

Full-Length Test 5 06/27/2003 12:17 PM Page 19

Page 20: MCAT Full Length5

58. What is the intensity at point A?

A. 0

B. I0/4 where I0 is the intensity of the unpolarizedlight incident on P1

C. I0/2, where I0 is the intensity of the unpolarizedlight incident on P1

D. I0, where I0 is the intensity of the unpolarizedlight incident on P1

59. If the transmission axes of P1 and P2 are perpendicu-lar to each other, what would be the effect of insert-ing a third linear polarizer at point A with itstransmission axis at a 45° angle with respect to thoseof the other polarizers?

A. The intensity of the light beam at point B wouldincrease.

B. The intensity of the light beam at point B woulddecrease.

C. The intensity of the light beam at point B wouldnot change, but its polarization would change.

D. The polarizer would have no effect on the inten-sity or the polarization at point B.

60. Which of the following represents the Brewster anglefor the light in Figure 1?

A. 90°

B. 0°

C. θtD. 90° – θt

20

GO ON TO THE NEXT PAGE.

Full-Length Test 5 06/27/2003 12:17 PM Page 20

Page 21: MCAT Full Length5

Passage X (Questions 61–66)

On a spherical planet of mass M and radius R, the accel-eration due to gravity is g = GM/R2, where G is the univer-sal gravitational constant. The magnitude of the escapevelocity from the surface of the planet is vesc = 2G M/R.This is the minimum velocity needed to escape the pull ofthe planet’s gravity.

Depending on the temperature, mass, and size of aplanet, many particles in its atmosphere may have speedslarge enough to escape into interstellar space. Even on theEarth, certain gases which are common to the Sun andouter planets have long ago “leaked away” into interstellarspace. This is because at a given temperature, particleswith greater speeds leak away more rapidly.

The probability P(v) that a gas particle has speed v isshown in Figure 1, for a gas at two different temperatures.This is known as the Maxwell speed distribution. Noticethat the probability goes to zero as the speed goes to infin-ity. As the temperature increases, high speed particlesbecome more probable. For an ideal gas of particles inthermal equilibrium at temperature T, the root-mean-square speed per particle is 3kBT/m, where kB is a con-stant equal to 1.38 10–23 J/K and m is the mass of aparticle. Since kinetic energy is 1/2 mv2, the average trans-lational kinetic energy per particle is 3/2 kBT. (Note:Assume that all of the gases referred to in the questionsbehave like ideal gases.)

Figure 1 Maxwell speed distribution of particles of a gas attemperatures T1 and T2 where T1 < T2

61. The mass of Jupiter is about 363 times that of theEarth, and Jupiter has a diameter 11 times as great.Approximately, what is the ratio of the magnitude ofthe escape velocity on Jupiter to that on Earth?

A. 3/11B. 11/3

C. 33D. 113

62. What is the minimum kinetic energy a particle ofmass m must have to escape from the surface of theEarth?

A. GmME/RE2, where ME is the mass of the Earth

and RE is the radius of the Earth

B. GmME/RE, where ME is the mass of the Earthand RE is the radius of the earth

C. 3/2 kBT, where T is the temperature at the sur-face of the Earth

D. 1/2 mvrms2, where vrms is the root-mean-square

speed of the particle

63. Consider nitrogen and oxygen gas at the same tem-perature. Which of the following statements is mostcorrect?

A. The maximum speed of the oxygen molecules isgreater than that of the nitrogen molecules.

B. The average translational kinetic energy of theoxygen molecules is equal to that of the nitrogenmolecules.

C. Each oxygen molecule has about 4/3 moretranslational kinetic energy than each nitrogenmolecule.

D. The friction between the oxygen and nitrogenmolecules raises the temperature of the mixture.

v

T2

T1

T1 < T2

P(v

)

21

GO ON TO THE NEXT PAGE.

Full-Length Test 5 06/27/2003 12:17 PM Page 21

Page 22: MCAT Full Length5

64. Although the composition of the Earth’s atmosphereis mostly stable over human lifetimes, compositionshifts do change over billions of years. Neglectingshifts due to climate and ecology, one would expectthat the ratio of:

A. hydrogen to oxygen will decrease.

B. water vapor to ammonia will decrease.

C. nitrogen to oxygen will increase.

D. methane to carbon dioxide will increase.

65. Assuming the Moon once had an atmosphere, thebest explanation for why the Moon currently has noatmosphere is:

A. gaseous elements never existed on the Moon.

B. gaseous elements escaped from the Moonbecause the temperature on the Moon is too highfor its size and mass.

C. gaseous elements escaped from the Moonbecause the temperature on the Moon is too lowfor its size and mass.

D. the magnetic field of the Moon is too weak.

66. The fact that the Earth is rotating about its polar axisaffects the escape velocity from the surface of theplanet. Taking into account the Earth’s rotation, theescape velocity at the North Pole is:

A. greater than the escape velocity at the SouthPole.

B. less than the escape velocity at the South Pole.

C. greater than the escape velocity at the equator.

D. less than the escape velocity at the equator.

22

GO ON TO THE NEXT PAGE.

Full-Length Test 5 06/27/2003 12:17 PM Page 22

Page 23: MCAT Full Length5

Passage XI (Questions 67–72)

Water that contains cations capable of forming precip-itates is defined as hard. Such cations include Mg2+, Ca2+,and Fe2+; all of which are capable of forming slightly sol-uble carbonate salts. These carbonate salts can build-up inthe water supply as boiler scale—solid deposits that canblock water pipes if not removed accordingly.

The concentration of carbon dioxide in water can affectthe precipitation of salts such as calcium carbonate. Dis-solved carbon dioxide reacts with water to form carbonicacid (H2CO3) which in turn dissociates to form a bicarbon-ate ion (HCO3

–) and a proton (Reaction 2). This bicarbonateion can react with Ca2+ to form a precipitate (Reaction 5), ordissociate further to form carbonate (Reaction 3). The car-bonate ion can also react with Ca2+ to form a precipitate(Reaction 4).

CO2(g) CO2(aq) ∆H = –19.41 kJ/mol

Reaction 1

CO2(aq) + H2O(l) H2CO3(aq) H+(aq)+ HCO3

–(aq)

Reaction 2

HCO3–(aq) H+(aq) + CO3

2–(aq)

Reaction 3

Ca2+(aq) + CO32–(aq) CaCO3(s)

Reaction 4

Ca2+(aq) + 2HCO3–(aq) → CO2(g) + H2O(l) + CaCO3(s)

Reaction 5

Table 1 gives relevant data for various species in Reac-tions 2 through 5.

Ka Kb Ksp

HCO3–(aq) 5.61 10–11 2.33 10–8 —

H2CO3(aq) 4.3 10–7 — —

CO32–(aq) — 1.79 10–4 —

CaCO3(s) — — 4.5 10–9

Table 1

67. Based on the information in the passage, HCO3– is:

A. a strong acid and a weak base.

B. a weak acid and a weak base.

C. a strong acid and a strong base.

D. a weak acid and a strong base.

68. Assuming that H2CO3 only dissociates into HCO3–

and H+, what is the pH of a 0.023 M H2CO3 solution?

A. 1.0

B. 2.0

C. 4.0

D. 8.0

69. A reservoir contains water saturated with both Mg2+

and Ca2+ ions. Which carbonate salt will constitutemost of the precipitate? (Note: Ksp(MgCO3) = 1.6 10–6.)

A. CaCO3, because its Ksp is smaller than that ofMgCO3.

B. MgCO3, because its Ksp is larger than that ofCaCO3.

C. There will be an equal mixture of salts.

D. Neither MgCO3 nor CaCO3 will precipitate.

23

GO ON TO THE NEXT PAGE.

Full-Length Test 5 06/27/2003 12:17 PM Page 23

Page 24: MCAT Full Length5

70. Which of the following could be added to a solutionof H2CO3 to make an acidic buffer?

A. H2SO4B. Na2SO4C. NaHCO3D. NaCl

71. According to the information in the passage, will anincrease in pH result in increased precipitation ofCaCO3?

A. Yes; increasing the pH shifts the equilibria ofReactions 2 and 3 in favor of carbonate ionproduction.

B. Yes; increasing the pH increases the proton con-centration and shifts the equilibria of Reactions2 and 3 in favor of carbonate ion production.

C. No; proton concentration does not affect theequilibria of Reactions 2 and 3.

D. No; increasing the pH shifts the equilibria ofReactions 2 and 3 in favor of H2CO3 production.

72. Which of the following factors will affect the molarsolubility of CO2(g) in a reservoir?

I. The pH of the water

II. The temperature of the water

III. The partial pressure of CO2(g)

A. I only

B. II only

C. I and III only

D. I, II and III

24

GO ON TO THE NEXT PAGE.

Full-Length Test 5 06/27/2003 12:17 PM Page 24

Page 25: MCAT Full Length5

Questions 73 through 77 are NOT based ona descriptive passage.

73. A 2-kg object slides down an incline with an angle of30° to the horizontal. If the force of friction is 2-N,what is the acceleration of the object?

A. 9.8 m/s2

B. 8 m/s2

C. 5 m/s2

D. 3.9 m/s2

74. What is the most stable outer electron configurationof Cr0?

A. 4s24p4

B. 4s23d4

C. 4s03d6

D. 4sl3d5

75. What is the pH of a 0.1 M aqueous NH3 solutionwhen 1% of ammonia undergoes hydrolysis?

A. 3

B. 7

C. 11

D. 13

76. Which of the following will not form an ionic crystal?

A. MgF2B. CO2C. NaCl

D. KNO3

77. What percentage of a sample of a radioactive elementwith a half-life of 4 days remains after 16 days?

A. 4%

B. 6.25%

C. 8%

D. 12.5%

STOP. IF YOU FINISH BEFORE TIME IS CALLED,CHECK YOUR WORK.YOU MAY GO BACK TO ANYQUESTION IN THIS SECTION ONLY.

25

Full-Length Test 5 06/27/2003 12:17 PM Page 25

Page 26: MCAT Full Length5

Full-Length Test 5 06/27/2003 12:17 PM Page 26

Page 27: MCAT Full Length5

Verbal ReasoningTime: 85 MinutesQuestions 78–137

DO NOT BEGIN THIS SECTION UNTIL YOU ARE TOLD TO DO SO.

Full-Length Test 5 06/27/2003 12:17 PM Page 27

Page 28: MCAT Full Length5

Passage I (Questions 78–83)

Neoconservatives increasingly blame our problems:…maleducation, victimism, criminal behavior, whatever...on the 1960s, as if in those years America abruptly plungedfrom Apollonian civilization to Dionysian chaos.Recently, the date when the U.S., or more precisely whenmany people within it, began to tip off the emotional trackswas fixed as August 1968. Supposedly, our national lifeunraveled when protesters battled Chicago police and left-ist intellectuals condoned them...

Violence of all kinds, and excuses for it, was multiply-ing well before 1968. More to the point, the Chicagodemonstrators—and many murderers, rapists, muggersand rioters—were born in the '40s and early '50s, andraised in the Eisenhower years, supposedly an era ofwholesome nuclear families informed by the right values.If we accept the premise of “family values” advocates, thatthe child is father to the man, then the '60s mayhem provesthat something was wrong with millions of American fam-ilies in the '50s, and that, moreover, the foundations of civ-ilized life were crumbling long before Chicago...

The ideas informing the much-maligned ‘60s leftistintellectuals (“Progressive” educators, and prophets ofpermissiveness like Dr. Spock) were already well-entrenched... Even before World War I, many influentialintellectuals had rejected the Protestant ethic of self-restraint. American capitalism’s shift to mass consump-tion, already established in the 1920s, undermined thatethic for the whole society... Rebellion against restraint,including self-restraint, was central to the Romantic move-ment. Glorification of spontaneity and the “noble savage”unsullied by civilization started with the Enlightenment, asdid the rise of determinism, which rationalizes misconductand denies personal responsibility... In 1897, while Godwas in His church, Queen Victoria was on her throne,America was on the gold standard, and all was right withthe world; socialism, anarchism and pragmatism weregaining popularity; Darwinism and higher criticism were

already undermining religion; Western high culture wasrotten with fin de siecle Decadence, inspired by, amongothers, the Marquis de Sade; and John Dewey wrote “MyPedagogic Creed,” launching American education on theroad to ruin.

But subversive intellectuals and cultural influencescould not have succeeded unless they were telling peoplewhat they wanted to hear. Ultimately, the blame for ourlack of “guardrails” lies with flawed human nature. Wesinners have our choices. The buck stops with the sinner,not with the tempter. Moreover, subversives were crashingan open door. Authority flopped at defending the civiliza-tion it guarded. So did such idols of neoconservatism’spantheon as...the family. Authority not only neglected theguardrails, or actually removed them, it set a disastrousexample by crashing them...

Which “family values” dominated the much-lauded‘50s? For the first time, entertainment and indulgence ofchildren was an unofficial national priority, from Disney-land and allowances to cars given upon graduation fromhigh school. Intensely materialistic, their own childhoodsharrowed by the Depression, fatally equating indulgencewith love, too many American parents were as far removedfrom the Romans who told their sons to return with theirshields or on them as the Keystone Cops are from the Cold-stream Guards. If their kids got into trouble at school orelsewhere, they typically sided with them and tried to bailthem out. They put comfort over character and raised theirchildren to fit in, be popular, make money, and have fun.

During this same time, the much-bewailed illegitimacyrate had already more than tripled between 1940 and 1960,from 7.1 per thousand unmarried females of childbearing ageto 21.8. Playboy appeared in 1953; presently, Elvis lewdlybucked his pelvis against his guitar to cheers from middle-class audiences; and in 1959, a federal judge ruled that “LadyChatterley’s Lover” wasn’t obscene, and that sending itthrough the mail was legal, opening the door to torrents offabricated satyriasis... Righteousness ran a poor second.

28

GO ON TO THE NEXT PAGE.

VERBAL REASONING

DIRECTIONS: There are nine passages in this VerbalReasoning Section. Each passage is followed by sev-eral questions. After reading a passage, select the onebest answer to each question. If you are not certain ofan answer, eliminate the alternatives that you know tobe incorrect and then select an answer from theremaining alternatives. Indicate your selection byblackening the corresponding oval on your answerdocument.

5

10

15

20

25

30

35

40

45

50

55

60

65

70

75

Full-Length Test 5 06/27/2003 12:17 PM Page 28

Page 29: MCAT Full Length5

Today’s lack of “guardrails” isn’t the '60s fault. Nogreat nation goes to pieces so abruptly as that. The '60s didnot cause our moral collapse. They only unmasked it. Butthe worst aspect of slapdash neoconservatives’ penchantfor blaming the '60s is that it understates the problem. Theassault on the beliefs underpinning civilized life, and thefailure to uphold them, have been going on not for 30 yearsbut for centuries...

78. Although not explicitly stated, the author wouldagree that nontraditional, free-thinking behaviorbegan:

A. in the 1950s with the proliferation of automo-biles, dating, teenage pregnancy, and musicians’lewd lyrics and gestures.

B. in the late 1800s because of mass destruction ofsocietal infrastructure from the Civil War.

C. centuries ago with movements such as theEnlightenment and determinism.

D. during the Great Depression, as individualswere forced to forgo concerns or proper societalbehavior and concentrate on the more basicneeds of food and shelter.

79 In which of the following would one most reasonablyexpect to find this passage?

A. a graduate sociology thesis

B. the editorial page of a scholarly journal

C. a commentary in a history textbook

D. a speech

80. If spoken to the author, the statement, “Our govern-ment makes no sense unless it is founded in a deeplyfelt religious faith—and I don’t care what [that faith]is” would be regarded by the author as:

A. a fundamental tenet of healthy democracy.

B. alien to neoconservative beliefs.

C. indicative of staggering intellectual vacuity.

D. an essential doctrine of “Progressive educators.”

81. The author cites all of the following as contributorsto societal “moral collapse” (line 78) EXCEPT:

A. the Marquis de Sade.

B. Darwinism.

C. violence.

D. lax parental discipline.

82. Which of the following statements, if true, wouldmost weaken the author’s argument?

A. The illegitimacy rate prior to 1940 was alwaysbelow 0.71 percent.

B. Only a small percentage of children raised in the1950s achieved great financial wealth duringtheir lifetimes.

C. Other nations have maintained traditionalmorality and stemmed societal violence despiteexposure to pornography, capitalistic mass con-sumption, and free thought.

D. Crime rates in western society have risen ininverse proportion to mean income levels andreligious devotion.

83. By stating,”...all was right with the world...”(lines35–36), the author implies that:

A. no dissenting philosophies were followed bygreat masses.

B. although global society was at complete peace,some dissension existed.

C. societal leaders correctly viewed western soci-ety as properly structured.

D. conservatives were naive in believing a utopiansociety existed.

29

GO ON TO THE NEXT PAGE.

80

Full-Length Test 5 06/27/2003 12:17 PM Page 29

Page 30: MCAT Full Length5

Passage II (Questions 84–90)

Some blacks had made livings as entertainers as farback as the eighteenth century, but the tradition of theblack entertainer really began after the Civil War, whenblacks were free to travel and set up businesses of theirown. They began by offering minstrel shows in imitationof the ones white entertainers had been presenting sincethe 1830s; when the minstrel show gave way to the varietytheater in the 1880s, many black dancers, singers, andcomics went along with it. By the 1890s, black entertain-ers were an accepted part of show business.

Nor was it just entertainers: By the end of the centuryblack writers and composers were beginning to create full-dress musical comedies for Broadway theaters. Theseshows frequently included white acts and , perhaps moresurprisingly, played to mixed audiences, with the blacksseated either upstairs or in half the orchestra sets. Indeed,by 1910, there had grown up a vogue for black shows writ-ten and directed by men like Rosamond Johnson, hisbrother James Weldon Johnson, Will Marion Cook, andWill Vodery, two of whom would act as tutors to DukeEllington...

Black show business had its ups and downs; accordingto Thomas Laurence Riis, who has made a careful study ofthe early black musical theater, these black shows at timeshad trouble getting bookings in the best houses and wereforced into second-tier ones, which some refused to play.

Then, in 1921, a musical called Shuffle Along, cookedup by the black entrepreneurs Flournoy E. Miller andAubrey Lyles, with music by Eubie Blake and NobleSissle, became a smash hit. The immensely popular song,“Memories Of You,” came from that show. JosephineBaker was in the chorus, and it skyrocketed FlorenceMills, on whose death [Duke] Ellington would compose atribute, to fame. The show started a renewed interest inblack entertainment that would become a boom by the endof the decade. According to J.A. Jackson, a black colum-nist who reported regularly on black entertainment forBillboard, the show “began the renaissance of the Negro inmusical comedy.” As early as 1923, Jackson would write,“Things are picking up very perceptibly for the coloredgroup in this industry due to the increase in the numbers ofpictures being made in the territory adjacent to New York.Another factor in the matter is that directors are desirous ofhaving naturalness and have just about eliminated themadeup, white actor in Negro characterizations.”

With the success of Shuffle Along and the dozens ofblack shows that followed over the decade, cabaret man-agers began booking the black acts in white locations. By

1923 Florence Mills was the headliner at the PlantationRoom on Broadway; in 1924 the jazz dancer Johnny Hud-gins was at the Winter Garden; and in 1923, a group ofyoung musicians from Washington would make it to Hol-lywood.

84. Based on the information in the passage, which of thefollowing statements is NOT true?

A. The musical Shuffle Along recharged America’sinterest in black musical theater.

B. Duke Ellington was taught by the most talentedmusicians of the early 1920s.

C. Black artists often accepted bookings in second-tier venues.

D. Black artists gradually gained acceptance asentertainers in the nineteenth century.

85. Which of the following statements, if true, wouldmost strongly support the author’s assessmentregarding the impact of Shuffle Along?

A. After Shuffle Along, many black artists werebooked into mixed-audience clubs.

B. The songs written for Shuffle Along are stillpopular today.

C. Black artists experienced a renaissance in the1920s.

D. Both black and white audiences enjoyed blackentertainers in Shuffle Along.

86. According to the passage, in which decade did blackartists achieve the greatest degree of integration intheater?

A. The 1880s

B. The 1920s

C. The 1890s

D. The 1830s

30

GO ON TO THE NEXT PAGE.

5

10

15

20

25

30

35

40

45

50

Full-Length Test 5 06/27/2003 12:17 PM Page 30

Page 31: MCAT Full Length5

87. The author suggests that black artists’ resurgent pop-ularity in the 1920s was primarily due to:

A. the freedom to travel and operate business afterthe Civil War.

B. audiences’ unpopular attitudes toward whiteactors in heavy makeup.

C. white audiences’ desire to see black performers.

D. an increase in film-making around New York atthe same time.

88. The author mentions Josephine Baker (lines 31–32)in order to:

A. underscore Shuffle Along’s importance to thedevelopment and dispersal of black theater andblack performers.

B. support his opinion that the success of ShuffleAlong lies in its talent.

C. draw attention to a black artist made famous byShuffle Along.

D. provide evidence of the popularity of blackartists to white audiences.

89. The author makes all of the following assertionsEXCEPT:

A. by the end of the 1920s, the black musical the-ater found renewed, widespread acceptance.

B. black entertainers first succeeded by imitatingwhite entertainers.

C. full-dress musical comedies were the soleprovince of white artists until the decade afterthe Civil War.

D. the black musical renaissance was not limited toblack audiences.

90. The passage discusses all of these critical points inthe history of black entertainment EXCEPT:

A. the creation of musical comedies by blackartists.

B. the development of the black entertainer inresponse to the Civil War.

C. the immediate consequences of the success ofShuffle Along.

D. the role of the black entertainer in the eighteenthcentury.

31

GO ON TO THE NEXT PAGE.

Full-Length Test 5 06/27/2003 12:17 PM Page 31

Page 32: MCAT Full Length5

Passage III (Questions 91–96)

Major wars often provide the punctuation marks of his-tory, primarily because they force drastic realignments inthe relationships among states. To this rule the First WorldWar was no exception. Long before the fighting ceased inNovember 1918, it was evident that the map of Europemust be redrawn and that reallocation of colonies, creationof a new international organization, and change in the eco-nomic balance must considerably affect the rest of theworld as well. The First World War heralded the end ofEuropean dominance, as the true victors in this predomi-nantly European war were America and Japan: two non-European powers. The European victors were bled whiteand suffered a pyrrhic victory from which none of themever really recovered. While this fact was not evident atthe war’s end, it was clear that the forthcoming settlementmust far exceed in geographic scope and complexity thoseother periodic realignments of the power balance, the 1648Treaty of Westphalia and the 1815 Final Act of Vienna, towhich it is often compared. Nobody doubted the magni-tude of the task ahead but nobody was properly preparedto undertake it.

As often happens, the sudden collapse of the enemytook the victors by surprise. Germany had been expectedto hold out until mid-1919, and in the autumn of 1918,Allied energy was more concentrated upon winning thewar than upon planning the peace. True, some planningwas in progress, but not always in the most effectual quar-ters. In the final year of the war, the smaller Allied statespursued their limited, specific aims with energy, butachieved only cautious and qualified commitments. Exileorganizations representing ethnic groups within the Cen-tral powers did the same with similar results. They recog-nized that the ultimate court of appeal would consist ofBritain, France, and America, but these three, who had thetask of planning for much of the world, were also respon-sible for winning the war. Not surprisingly, that came first.

91. Based on the information presented in the passage,the author would most likely agree with which of thefollowing statements?

A. Britain, France, and the United States were pre-pared for the rapid demise of Germany.

B. European states never really recovered from therealignment of power that followed the 1815Final Act of Vienna.

C. Europe’s dominance of world politics endedwith World War I.

D. Creation of a new international organizationpreoccupied the minds of the victorious powersbefore the war was won.

92. In the context of the passage, the phrase “pyrrhic vic-tory” (line 13) means:

A. a total defeat.

B. an overwhelming victory.

C. a costly triumph.

D. a disastrous truce.

93. If true, which of the following arguments would mosteffectively weaken the author’s argument?

A. The European powers eventually recoveredfrom World War I.

B. Without intervention, the economic balance ofEurope stabilized rapidly after the war ended.

C. The 1648 Treaty of Westphalia was even morecomplex than the agreement that ended WorldWar I.

D. World War I caused no drastic change inEurope’s central importance to global politics.

32

GO ON TO THE NEXT PAGE.

5

10

15

20

25

30

35

Full-Length Test 5 06/27/2003 12:17 PM Page 32

Page 33: MCAT Full Length5

94. The passage suggests that the United States, Britainand France:

A. were largely responsible for the direction andaims of the peace process.

B. could have ended the war much sooner.

C. doubted the peace process would take very long.

D. paid close attention to the claims of smallerAllied nations in the late stages of the conflict.

95. The second paragraph suggests that:

A. the early plans for peace were mostly made byineffectual leaders.

B. Allied forces were intently focused on winningthe war in the fall of 1918.

C. Germany’s war effort was expected to last intothe 1920s.

D. wars run a natural, predictable course.

96. Based on the information in the passage, which of thefollowing is NOT true?

A. Ethnic factions within the Central Powers wononly cautious promises from the Allied nations.

B. Germany’s early collapse in 1918 found theAllied nations somewhat unprepared for peace.

C. The Allied nations knew the peace processwould be a monumental task.

D. The reallocation of colonial possessions was notconsidered before the peace process began.

33

GO ON TO THE NEXT PAGE.

Full-Length Test 5 06/27/2003 12:17 PM Page 33

Page 34: MCAT Full Length5

Passage IV (Questions 97–103)

In Prisoners Of Men’s Dreams, Suzanne Gordonargues that American feminism has lost sight of its originalgoal of transforming the world into a kinder, gentler place.Gordon deplores the sort of feminism that has triumphedinstead: cold, ruthless, “equal-opportunity” feminism,which aims for women’s entrance into the masculine pub-lic world and their achievement by male standards ofexcellence.

The heart of the book consists of excerpts from a hun-dred interviews with career women, who do a lot of com-plaining about fatigue and disillusion. At the end, Gordoncalls for a National Care Agenda that would make “caregiv-ing” rather than competition the ultimate American value.

Suzanne Gordon is obviously an intelligent, sympa-thetic, and well-meaning person, but Prisoners Of Men’sDreams is a good example of the kind of sentimental,unlearned effusion that has become a staple of contempo-rary feminism and that most men rightly ignore.

Like so many American feminists, Gordon is com-pletely out of her depth as a social analyst. Awkward, unin-tegrated quotes from Adam Smith and Woodrow Wilsonare waved around to disguise her lack of familiarity witheconomics, history, and political science.

We are presented with the usual three-handkerchieftear-jerker scenario about Big, Bad, Ugly America—thatcorrupt, empty, greedy society which all those wonderful,warm, benevolent people around the world look at withdisgust. This point of view is the essence of chic these daysamong know-nothing feminists and the preening pseudo-leftists who crowd our university faculties.

Well, let me tell you: As a child of Italian immigrants,I happen to think that America is the most open, dynamic,creative nation on God’s green earth. As a scholar, I alsoknow that it is capitalist America that produced the mod-ern independent woman. Never in history have women hadmore freedom of choice in regard to dress, behavior,career, and sexual orientation.

Gordon’s insistence on defining women as nurturantand compassionate drove me up the wall. My entire rebel-lion as a child in the Fifties was against this unctuous,preachy stuff coming from teachers, nuns, and Girl Scoutleaders.

Gordon’s caregiving “transformative feminism” is justas repressive and reactionary as the “patriarchy” it claimsto attack. Minerva save us from the cloying syrup of coer-

cive compassion! What feminism does not need, it seemsto me, is an endless recycling of Doris Day Fifties clichesabout noble womanhood.

97. The author of the passage would most likely claimthat someone who did NOT agree with her view offeminism was:

A. independent.

B. sentimental.

C. rebellious.

D. matriarchal.

98. According to the author, which of the followingstatements is NOT true?

A. Works like Prisoners of Men’s Dreams are rou-tinely ignored.

B. Gordon’s National Care Agenda is well-meaning.

C. Gordon has experience as a social analyst.

D. Gordon’s feminist rhetoric equates womanhoodwith nurture and care.

99. The main argument of the passage is that:

A. Gordon is an intelligent woman.

B. Gordon finds America repressive, corrupt andgreedy.

C. Gordon’s argument reinforces outdated feministstereotypes.

D. Doris Day is not a good feminist model.

34

GO ON TO THE NEXT PAGE.

5

10

15

20

25

30

35

40

45

Full-Length Test 5 06/27/2003 12:17 PM Page 34

Page 35: MCAT Full Length5

100. In the passage, “transformative feminism” (line 43)is used to mean:

A. a political agenda with caregiving as its guidingprinciple.

B. the process of reinventing power structures toprovide equal opportunities for women.

C. encouraging people to adopt “womanly” virtuesas cultural standards.

D. a goal of redefining feminine roles to includenurturing and compassion.

101. Which of these assumptions is NOT implicit in theauthor’s view?

A. Stereotypes concerning female attributes areincorrect.

B. “Equal-opportunity” feminism is fundamentallyideal.

C. Gordon’s rationale gives credence to Fifties-style cliches of feminine roles.

D. American values and mores are considerablymore liberal than those of other nations.

102. The author’s most strenuous objection to Gordon’sthesis is that:

A. it lacks scholarly rigor.

B. it offers political solutions for moral questions.

C. it depicts modern feminism as cold and ruthless.

D. the modern independent woman is free tochoose her dress, career, and behavior.

103. The author’s opinion of Gordon’s view of Americanvalues can best be described as:

A. disinterested.

B. tolerant.

C. uninformed.

D. dismissive.

35

GO ON TO THE NEXT PAGE.

Full-Length Test 5 06/27/2003 12:17 PM Page 35

Page 36: MCAT Full Length5

Passage V (Questions 104–111)

…Welles had directed himself many times on thestage, but somehow the very permanence of film, the pos-sibility of displaying one version of a role was somewhatformidable to him. On the stage he would—and did—change his approach to a part almost from night to night tobetter hone it into shape and to sense what an audience, aparticular audience, needed or wanted from any given per-formance. Once the curtain went up, the performance’ssuccess or failure was the actor’s own: he had no directorto say “Cut!” if he was bungling his lines or losing hisaudience. This fluidity was impossible when working withthe terminal, packaged quality of a finished motion pic-ture. The camera never laughed, nor did it fall asleep. Itwas impossible for it to gauge one’s performance.

Chaplin had been successful in directing himself onfilm, but there were two important differences betweenwhat he had accomplished and what Welles was attempt-ing to do: Until that time, Chaplin had worked exclusivelywith silent film, and all his films were comedies. Althoughit is true that drama did not contain the demands of pacingfound in comedy, the dimension of spoken dialogue couldadd enormous problems for someone appraising himself.Welles, however, was choreographing Citizen Kane withthe unhesitating measure and continual rhythm of an up-tempo musical comedy or, as someone has since sug-gested, a grand opera.

So that he could see how he would ultimately moveamid and through a particular set and piece of business, healways had on hand a “double,” not in the traditional actingsense of someone who looked like him, but an actor whowas as nearly exact in his height, weight, and general bod-ily structure as possible. Sometimes he had William Alland,although smaller and thinner, walk through his part for him.When the young and eager Kane bounced out of the horse-drawn carriage and into the building of the New YorkInquirer, for instance, it was Welles’ double who first wentthrough the motion without benefit of camera, those thatWelles the actor would ultimately make on the screen. Thatwas the easy part. More difficult was going through the ges-tures himself, first in rehearsal, and then for the initialshootings. How is one to know that a slight trembling of thelips, or the blink of the eye, or an awkward movement ofthe shoulder, will not be recorded by the camera? He foundit difficult hitting his own chalk marks, that predeterminedplace where he was to stand so that [the camera] could bet-ter work [its] magic. On the stage he could accidentally, orby choice, be slightly more stage left or right, but with film,although he still had to maintain rhythm and endurancewith body movements, it was imperative that he hit thesemarks accurately and with absolute precision every time.

It took only a few sessions of directing himself andothers, followed by a careful examination of the rushes, toprove how intrinsically different stage and screen actingreally were. The lesson of the whole history of the art ofacting in films began to wash over him: the fact that someof Eisenstein’s best actors had never seen the inside of atheater; that some of the greatest stage performers—Bern-hardt and Duse and Beerbohm Tree among them—weredisappointments on the screen.

104. The main idea of the passage is that:

A. Welles approached the challenge of directinghimself on film in exactly the same manner ashe had done on stage.

B. Welles improvised techniques, such as using adouble, to free himself to examine his stagingtechniques.

C. Welles found only through experience that filmdirecting is quite different than stage directing.

D. the great actors of the stage—Bernhardt, Duse,and Beerbohm Tree—were unable to translatetheir ability to film.

105. The author mentions Chaplin (lines 15–19) primarilyto:

A. compare Welles’ difficulties to those of anothergreat actor who had directed himself on film.

B. contrast Welles’ pacing of Citizen Kane to thepacing of a comedy film.

C. underscore the fundamental change that spokendialogue had brought into film-making.

D. introduce the notion that Welles’ choreographywas similar to that of a grand opera.

106. If true, which of the following would most weaken the author’s argument?

A. Welles quickly adapted his film-acting skills tothe stage.

B. Stage acting uses the same timing and place-ment techniques as screen acting.

C. Chaplin had worked in both silent films and“talkies” with great success.

D. The success of a stage production depends onthe interaction of the cast.

36

GO ON TO THE NEXT PAGE.

5

10

15

20

25

30

35

40

45

50

55

Full-Length Test 5 06/27/2003 12:17 PM Page 36

Page 37: MCAT Full Length5

107. The term “rushes” (line 52) means:

A. a careful sketch made from staging ideas.

B. spots marked on the stage to cue actors’ posi-tions.

C. unedited film footage.

D. changes made to the script by the director.

108. It can be inferred from the passage that Welles:

A. found that drama films are more difficult to pacethan comedies.

B. made his director’s job much more difficult bypacing Citizen Kane like an up-tempo musicalcomedy.

C. directed himself more successfully than didChaplin.

D. found it easier to gauge his performances onfilm than on stage.

109. According to the passage, the author would describeWelles’ stage acting technique as all of the followingEXCEPT:

A. fluid.

B. changeable.

C. formulaic.

D. responsive.

110. Which of the following is asserted with no support-ing evidence?

A. Some of the greatest stage actors were disap-pointments on the screen.

B. Welles used another actor to assume his partsduring the planning stages of his films.

C. As a stage director, Welles had more flexibilitythan he did in film work.

D. William Alland was not an exact physical dou-ble for Welles.

111. According to the passage, which of the following istrue about stage acting technique?

A. It denies the actor a certain freedom of expres-sion.

B. It promotes a fixed interpretation of primarycharacters.

C. It requires less physically exacting techniqueand timing than does film.

D. It translates easily to film.

37

GO ON TO THE NEXT PAGE.

Full-Length Test 5 06/27/2003 12:17 PM Page 37

Page 38: MCAT Full Length5

Passage VI (Questions 112–118)

A behavior as complex as sleep—with its highly dif-ferentiated component non-REM and REM phases—isunlikely to be dedicated to any one particular function, yetfolk wisdom has tended to collaborate with scientificreductionism in supposing that one, and only one, functionis served by sleep. The universal favorite candidate for thisfunction is rest. This is probably the carryover of the naivenotion that in sleep our brains are at a low and monotonouslevel of activity. Yet in spite of years of research, sciencehas not definitely established even one function for sleep.

It is at least logically appropriate to assume, as ourmothers did, that sleep is necessary for health since wesubjectively experience sleep as restful and restorative. Sooverwhelmingly clear is the sense of restoration followinga good night of sleep that no one in his or her right mindwould abandon the rest theory despite the deafeningsilence of physiology on this question. In this sense, thesubjective experience of sleep should more powerfullymotivate us to seek physiological or behavioral explana-tions, just as the experience of dreaming should have moti-vated scientists to look for evidence of brain activationduring sleep.

But the rest theory is likely to require a more special-ized answer in the case of the kind of sleep we humansshare with our fellow mammals. The reason is that restdoes not require sleep. Inactivity alone should suffice toprovide us with rest. And many organisms already spend agood deal of their wake-state time at rest.

The rest function may well be further elaborated inhigher animals with complex brains so as specifically torestore efficiency to such crucial wake-state functions asattention and memory. Such rest is likely to be associatedwith more active processes than the simple passive one therest theory would imply. For example, while the brain-mind is freed of the task of monitoring and rememberingnew information in sleep, it can review and reorganize itsown already acquired data. It is in this sense that most ofthe new hypotheses arising from modern sleep researchdistinguish themselves from Freud’s contributions andthose of his contemporaries and predecessors.

The reason it has been difficult to establish convincingfunctional hypotheses is, as usual, methodological. Themost obvious experimental approach to functional ques-tions is to deprive people of sleep and then observe anybehavioral deficits. But anyone who has undergone a nightof self-imposed sleep deprivation will know that thisapproach is not only painful but difficult. So many thingsneed to be done in order to keep oneself awake that it is

virtually impossible to control for nonspecific and unin-tentional effects of the deprivation procedure itself. Thesenonspecific procedural factors may cause the deficits inperformance. And despite all efforts to stay awake, wedoze off anyway!

Preferable to sleep deprivation would be some measureof brain function (or behavioral capability, or psychologi-cal process) that could be tested around the clock and befound to (1) deteriorate as the wake period is prolongedand (2) recover dramatically following an epoch of sleep.This positive functional model, in which sleep would beshown to reverse a process that declines progressively inwaking, has never been successfully applied to any func-tional question.

Even if one were successful in completely preventingsleep and effectively controlling the inadvertent sideeffects of deprivation, one would then have to move to amolecular or cellular neurobiological approach to under-stand the positive effects of such deprivation. Researchershave convincingly demonstrated that sleep loss can befatal. Sleep-deprived rats fail to regulate their energy andliterally consume themselves metabolically. Now the ques-tion is: How are such effects mediated? This exampleshows clearly that sleep deprivation per se is only aninstrumental tool; it is not an analytic probe. It may help usto ask the right question but it can never, by itself, give usthe answers we seek.

112. The passage suggests that sleep researchers’ inabilityto establish “convincing functional hypotheses” indi-cates that:

A. the hypotheses of Freud and his predecessorsmust still be considered potentially valid.

B. the theory that the purpose of sleep is to allowthe organism to rest must be eliminated.

C. researchers have experienced difficulty devisingan appropriate experimental approach.

D. sleep may not in fact have any particular func-tion aside from rest.

38

GO ON TO THE NEXT PAGE.

5

10

15

20

25

30

35

40

45

50

55

60

65

70

75

Full-Length Test 5 06/27/2003 12:17 PM Page 38

Page 39: MCAT Full Length5

113. Which of the following, if true, would most weakenthe author’s argument against the theory that the solefunction of sleep is simply to provide passive rest?

A. Folk wisdom and science seldom agree in theirexplanations of natural phenomena.

B. Selective deprivation of sleep during each of itscomponent phases has varying behavioraleffects.

C. Some species of mammals spend over 50% oftheir awake time at rest.

D. Measures of attention and memory remain sta-ble regardless of time spent in sleep.

114. On the basis of the author’s comments in the secondparagraph about our subjective experience of sleep, itis reasonable to conclude that the author believesthat:

A. subjective experience can stand alone as suffi-cient evidence to prove a theory about bodilyfunctions correct.

B. dreaming subserves some sort of physiologicalrestorative function.

C. a lack of scientific evidence for a theory doesnot automatically invalidate it.

D. subjective experience can always be relied uponas an accurate source of information.

115. Although no supporting evidence is provided, theauthor claims that:

A. the function of sleep cannot be determined sim-ply by observing the effects of sleep deprivation.

B. brain activity is at a low and monotonous levelduring sleep.

C. many organisms spend a good deal of theirwake-state time at rest.

D. sleep deprivation can be fatal.

116. Suppose that a medical resident, who must stay awakeduring work-shifts of 36 hours duration, is given asimple memory test at the beginning and the end ofone such shift. Her score is found to be significantlylower at the end of the shift. Which of the followingresponses is most consistent with the passage?

A. The experiment establishes that sleep depriva-tion affects memory function but doesn’t pro-vide information as to how this occurs on amolecular level.

B. The reason memory capacity decreased is thatthe resident was required to retain a lot of newinformation and wasn’t able to review and reor-ganize it during sleep.

C. Because many factors could have affected theresident’s memory capacity over the course ofthe shift, it cannot be concluded that the mem-ory deficit was the result of sleep deprivation.

D. The resident’s memory should have been testedduring the work-shift as well as before and after.

117. Which of the following would be the best example ofthe successful application of the author’s “positive func-tional model” (line 59) to a sleep-function question?

A. A subject’s mood is evaluated before and aftersleep deprivation and is found to have beenimproved by deprivation.

B. The overall level of brain electrical activity in asubject is measured over a 24-hour period and isfound to oscillate regularly.

C. The concentration of a neurotransmitter in asubject’s brain is found to decrease over thecourse of the day but is restored after sleep.

D. A subject’s reading speed is tested after a nightof sleep and is found to be nearly identical to theprevious day’s rate.

118. The author asks the question “How are such effectsmediated?” (line 71) in order to show that:

A. knowing which question is the right one to askis crucial.

B. the dearth of knowledge about sleep at a molec-ular level is staggering.

C. scientists cannot fully investigate sleep usingsleep deprivation.

D. productive scientific research always leads tofurther questions.

39

GO ON TO THE NEXT PAGE.

Full-Length Test 5 06/27/2003 12:17 PM Page 39

Page 40: MCAT Full Length5

Passage VII (Questions 119–124)

...Can the university, with its special trust of protectingfree speech, be hampered by the restrictions of civility?What kind of a frill is etiquette, anyway, for those in thenoble pursuit of truth?

These questions are raised whenever a loose-tonguedstudent turns publicly nasty. When Brown Universityrecently expelled such a student, many argued that allrestrictions of free speech are intolerable in the university.Brown’s President, Vartan Gregorian, agreed with thatpremise and neatly reclassified the offensive speech asbehavior.

But the premise is wrong.

The special trust of a university is not to foster unlim-ited speech. It is to foster unlimited inquiry. And totallyfree speech inhibits rather than enhances the free exchangeof ideas.

The law cannot restrict such speech without violatingour constitutional rights. But etiquette, the extra-legal reg-ulative system that seeks to avert conflict before itbecomes serious enough to call in the law, can and does.You may have a legal right to call your mother an idiot, orsomebody else’s mother a tramp, but you won’t if youknow what’s good for you.

Nor could you convince many people that the contro-versy that such remarks are likely to provoke will lead toadvances in knowledge.

The university needs to enforce rules banning speechthat interferes with the free exchange of ideas. It must pro-tect the discussion of offensive topics but not the use ofoffensive manners. It must enable people freely to attackideas but not one another.

Education is impossible without the order that preventsintimidation and mayhem. When children first enterschool, they must be taught to sit still, refrain from taunt-ing their classmates, show respect for their teacher andwait their turn to talk, or they will never be able to learn.

To those who find it horrifying that the universityshould allow a lesser degree of free speech than the lawpermits, it might be pointed out that the law itself restrictsfree speech in its pursuit of juridical truth. Try saying someof the things in a courtroom that the law will protect yourright to say in a barroom.

Jurisprudence uses etiquette in courtroom procedure,not only to restrict speech but to impose standards of dress,comportment and forms of address—matters over whichuniversities have long since abandoned authority.

Legislators and diplomats also know the value of keep-ing speech within the bounds of civility. The parliamentaryetiquette book, “Robert’s Rules of Order,” proscribes “dis-orderly words” and forbids speakers “to arraign themotives of a member” during strongly worded debate. “Itis not the man, but the measure, that is the subject ofdebate,” decrees it’s section on “Decorum.”

The rougher the conflict, the more manners are needed.Only when insults, harassment, disrespect and obscenityare banned can people engage in truly substantive argu-ment.

Of course it is also a personal insult to call someone aracist or a sexist. Incivility is no more acceptable indefense than in attack.

Rebuttal, however, is a staple of open debate. Membersof the university community should always have theopportunity to attack ideas—but not to attack people. Theuniversity should be obliged to provide a forum for anyonewho wants to argue for or against an idea, provided theargument is made in good faith and a polite manner.

The standard of academic etiquette must be requirednot only in the classroom and lecture hall but wherever thecommunity of scholars gathers—residence halls, diningcommons, recreational facilities. Invective, whether spo-ken or conveyed through posters or graffiti, in the class-room or in the community, is detrimental to rationaldebate, to which universities are dedicated.

40

GO ON TO THE NEXT PAGE.

5

10

15

20

25

30

35

40

45

50

55

65

70

Full-Length Test 5 06/27/2003 12:17 PM Page 40

Page 41: MCAT Full Length5

119. According to the essay, for what can you be expelledfrom a university?

I. Standards of comportment.

II. Offensive behavior.

III. Offensive speech.

A. II only

B. I and III only

C. III only

D. I, II and III

120. According to the passage, what is the strongestargument for the use of etiquette versus the firstamendment?

A. The use in our courtrooms

B. The use in parliament

C. Racist remarks not leading to advances inknowledge

D. Incivility being no more acceptable in defensethan attack

121. To what are the authors referring when they say, “Butthe premise is wrong” (line 12)?

A. Offensive behavior

B. The expulsion of a student on the basis of offen-sive speech

C. Intolerable restrictions of free speech in the uni-versity

D. Restrictions of civility hampering the university

122. The authors define “the university” as:

A. a forum dedicated to rational debate.

B. a forum that must protect the discussion ofoffensive topics.

C. an institution of higher learning.

D. a forum for invective.

123. According to the passage, in what location does thelaw protect your right to free speech?

A. In parliament

B. In residence halls

C. In a bathroom

D. In a barroom

124. In defense of restrictions on offensive speech, theFourteenth Amendment is often cited. It guarantees“equal protection of the laws” to all, including thosewho are targets of offensive speech. The authors’main response to this would probably be:

A. all restrictions of free speech are intolerable inthe university.

B. the law cannot restrict such speech without vio-lating our constitutional rights.

C. the university must protect the discussion ofoffensive topics but not offensive ideas.

D. reclassify the offensive speech as offensivebehavior.

41

GO ON TO THE NEXT PAGE.

Full-Length Test 5 06/27/2003 12:17 PM Page 41

Page 42: MCAT Full Length5

Passage VIII (Questions 125–132)

...[post-World-War-II Director of Policy PlanningGeorge F.] Kennan’s strategy had been to try to bring aboutchanges, over time, in the Soviet concept of internationalrelations: to convince Russian leaders that their interestscould be better served by learning to live with a diverseworld than by trying to remake it in their own image. Ken-nan had rejected both war and appeasement as means ofaccomplishing this; it could only be done, he thought,through a long-term process of what might be called“behavior modification”—responding positively to what-ever conciliatory initiatives emanated from the Kremlin,while firmly countering those that were not...

Kennan took the position that modifying Soviet behav-ior required both positive and negative reinforcement: itwas as important to reward the Kremlin for conciliatorygestures as it was to resist aggressive ones. This meantbeing prepared to engage in such negotiations that wouldproduce mutually acceptable results. The [Truman] admin-istration conveyed the appearance of being willing to dis-cuss outstanding issues with Moscow, but Kennanregarded several of its major actions between 1948 and1950...[among them] the formation of the North AtlanticTreaty Organization (NATO)...as certain to reinforceSoviet feelings of suspicion and insecurity, and hence, tonarrow opportunities for negotiations...

The initiatives for the North Atlantic Treaty came fromthe West Europeans themselves, and reflected the uneasi-ness they felt over the disparity in military power on theEuropean continent: the Russians had, at that time, thirtydivisions in Eastern and Central Europe alone; comparablecombined U.S., British, and French forces came to lessthan ten divisions.

Intelligence estimates credited the Russians with theability to sweep to the English Channel and the Pyreneesin a matter of weeks… By September 1948 theWestern Union countries (Great Britain, France, and theBenelux countries), together with the United States andCanada, had agreed on the outlines of a treaty providingthat an attack on any one of them or on any other nationsthat might be included within the terms of the treaty wouldbe regarded as an attack upon all.

Because of a trip to the Far East and a subsequentperiod of hospitalization, Kennan had not been involved inthe initial discussions regarding European security early in1948, but upon his return he quickly made clear his reser-vations about the course the administration had chosen tofollow. These boiled down to three points: (1) That theEuropeans had mistaken what was essentially a political

threat for a military one, and that they risked, as a result,“a general preoccupation with military affairs,to the detriment of economic recovery”; (2) that outsidethe immediate North Atlantic area, “which embraces a realcommunity of defense interest firmly rooted ingeography and tradition,” any alliance extended to onlysome countries would render the rest all the more vulnera-ble...; (3) that an alliance made up of [Western European]nations would amount to “a final militarization of the pre-sent dividing-line through Europe,” and that “no alteration,or obliteration, of that line could take place without havingan accentuated military significance.” Such a developmentmight be unavoidable, “but our present policy is stilldirected...toward the eventual peaceful withdrawal of boththe United States and the U.S.S.R. from the heart ofEurope...”

These were not isolated concerns. There was worry inWashington that emphasis on rearmament would delayrecovery; indeed, one of the conditions attached to theadministration’s military assistance program for WesternEurope was that economic revival would continue to havefirst priority. The question of how to include some coun-tries without appearing to write off others also caused agreat deal of agonizing: In the end, the administrationstretched the concept of “North Atlantic” to encompassItaly, but refused to extend it to Greece, Turkey, Iran or toform a comparable pact with the non-communist countriesof the Western Pacific. There was less concern about Ken-nan’s third point—that an alliance might freeze the exist-ing division of Europe—simply because most observersalready regarded that division, by mid-1948, as an accom-plished fact...

Despite its reservations, the administration went on toconclude a North Atlantic Treaty and initiate a program ofmilitary assistance to its members. Kennan came to see,regretfully, that [because of military insecurity of Euro-peans] there were few alternatives...

[The decision on NATO]...was not intended to pre-clude eventual negotiations with the Russians, but it didseek to defer them until requisite levels of “strength” hadbeen reached. In so doing, however, it left little room forefforts to alter the Soviet concept of international relationsthrough positive as well as negative reinforcement.

42

GO ON TO THE NEXT PAGE.

5

10

15

20

25

30

35

40

45

50

55

60

65

70

75

80

85

90

Full-Length Test 5 06/27/2003 12:17 PM Page 42

Page 43: MCAT Full Length5

125. The passage suggests that Kennan’s “behavior modi-fication” (line 10) approach to changing the Sovietconcept of international relations was:

A. unlikely to be successful if the Kremlin alwaysmade conciliatory gestures.

B. moderate in comparison with the approaches hedecided to reject.

C. a logical outgrowth of his extensive backgroundin behavioral psychology.

D. an extension of American strategy during theSecond World War.

126. Based on the passage, Kennan’s attitude aboutheightening Soviet insecurity and suspicion throughthe formation of NATO was one of:

A. belief that it was appropriate negative reinforce-ment of the Soviet military presence in Europe.

B. hope that it would prompt the Soviets to makemore conciliatory gestures.

C. disapproval because it would make modifyingSoviet international behavior much more difficult.

D. belief that negotiations with the Soviets wouldbe easier on a more even playing field.

127. The passage suggests that the impetus for the forma-tion of NATO was:

A. information that a Russian attack on WesternEurope was impending.

B. the understanding that no nation could with-stand a Russian attack without assistance.

C. the desire to aid the Western European eco-nomic recovery as well as to guarantee militaryassistance.

D. the fear that the Soviets would try to capitalizeon their military advantage.

128. Kennan assumed which of the following in makinghis first point (lines 47–52) about NATO?

A. The formation of the military alliance wouldspur economic growth.

B. The presence of the thirty Soviet divisions didnot mean they were going to attack.

C. The economic recovery in Europe had been pro-gressing slowly.

D. It’s always a mistake to make military affairs ahigher priority than economic affairs.

129. Kennan’s reaction to the administration’s refusal toextend NATO membership to Greece, Turkey or Iranwas most likely one of:

A. understanding, because these countries did nothave the same geographic defense interests asthe Europeans.

B. approval of the fact that the concept of “NorthAtlantic” was not over extended.

C. disappointment that those countries could notnow be employed in anti-Soviet strategy.

D. trepidation that these countries were now moreopen to potential enemy aggression.

130. Which of the following explains why the Trumanadministration was not worried about Kennan’sobjection that NATO would amount to “a final mili-tarization of the present dividing-line throughEurope”?

A. They believed that it would be possible to alterthe line through negotiations of peaceful with-drawal.

B. They wanted to maintain a strong Americanmilitary presence in Europe.

C. They felt it was too late to prevent the solidifi-cation of the dividing line.

D. Neither of Kennan’s other two objections toNATO had given them cause for concern.

43

GO ON TO THE NEXT PAGE.

Full-Length Test 5 06/27/2003 12:17 PM Page 43

Page 44: MCAT Full Length5

131. The passage suggests that, with regard to the reser-vations expressed by Kennan about NATO, theadministration was:

A. often in agreement but ultimately undeterred.

B. unresponsive to his proposals for improvement.

C. able to counter each of his criticisms.

D. forced to carefully re-examine its objectives.

132. Which of the following conclusions would be most inaccord with a theme of the passage?

A. Military alliances invariably have drawbacksthat render them ineffective.

B. Behavioral modification is the only way tochange a government’s concept of internationalrelations.

C. Coherent international strategy can flounderbecause of the military situation.

D. Negotiations should be conducted between twopowers once military equilibrium has beenestablished.

44

GO ON TO THE NEXT PAGE.

Full-Length Test 5 06/27/2003 12:17 PM Page 44

Page 45: MCAT Full Length5

45

GO ON TO THE NEXT PAGE.

Full-Length Test 5 06/27/2003 12:17 PM Page 45

Page 46: MCAT Full Length5

Passage IX (Questions 133–137)

The new field of molecular anthropology has providedimportant new data on primate evolution, but many pale-ontologists have been slow to accept them. They argue thatfossils are the only direct evidence and that molecules arenot relevant to the study of extinct species. With regard tohuman origins, fossil bones and teeth may appear to con-flict with molecular data, but, as we have seen withRamapithecus, the fossil evidence is often fragmentary andsubject to more than one interpretation.

The two approaches are quite different: Paleontologistsdig up bones from the earth, study their shapes, and com-pare them to other living and extinct species in order toestablish their place in an evolutionary phylogeny.

Molecular anthropologists study the proteins of livingspecies and deduce how long ago two species divergedfrom a common ancestor... Proteins are madeup of various combinations of the basic twenty aminoacids, arranged in definite sequences. A given protein mayinclude hundreds to thousands of amino acids. The pro-teins of closely related species, such as horse and donkeyor dog and fox, are nearly identical; whereas species thatdiverged more than 100 [million years ago], such as shrewand opossum, have many sequence differences.

These differences can be measured precisely, and theirnumber is approximately proportional to the divergencetime. Such “molecular clocks” are particularly valuable forevolutionary study because results can be, and have been,replicated in numerous laboratories, whereas the analysisof fossil bones and teeth is somewhat subjective, andagreement between researchers may be difficult toachieve.

Paleontological phylogenies have indicated a long,independent evolution for the five living hominid groups.They have suggested that Pliopithecus was an early gib-bon, that different species of Dryopithecus were ancestralto gorilla, chimpanzee, and orangutan, and that Ramap-ithecus was ancestral to the human line.

In contrast, the molecular phylogenies show a rela-tively recent divergence for all the apes, with the Asianapes splitting off earlier than the rest. This rules out thepossibility that Pliopithecus is an ancestral gibbon or Dry-opithecus or Proconsul are ancestral apes. The three-waydivergence between human, chimpanzee, and gorilla about5 [million years ago] does not preclude Australopithecusas an early representative of the human family. So, asidefrom the equivocal fossil evidence itself, the molecularfindings make it even less likely that Ramapithecus could

have been a human ancestor.

To consider Ramapithecus as hominid, one would haveto assume that primate proteins have evolved at half therate of shark, fish, frog, snake, kangaroo, mouse, and ele-phant proteins. Some have argued that proteins evolvemore slowly in animals with more time between genera-tions—a presumption refuted by the evidence that mouseand elephant proteins have evolved at the same rate, ashave loris and human proteins. This external check on thestatistical constancy of the molecular clock is further sup-ported by internal evidence: numerous proteins with dif-ferent rates of change (cytochrome c, albumin, transferrin,hemoglobin, and histones) indicate similar divergencetimes. That is like timing the same event with an hourhand, minute hand, and second hand and finding out thatthe times come out the same. The statistical constancy ofthe molecular clock is not an assumption but an observa-tion based on an enormous amount of data.

From the biochemical evidence, then, there was butone ancestor prior to the ape-human split, not three differ-ent ones, for humans and the African apes. The molecularinformation had made it clear that we cannot look athuman origins as an isolated phenomenon, but must con-sider the event as part of the radiation that included theAfrican apes. If Ramapithecus has a place in human ances-try, it will have a place in chimpanzee and gorilla ancestryas well.

133. The author would agree with which of the followingstatements regarding the origins of the five livinghominid groups?

I. Pliopithecus was an early gibbon.

II. Dryopithecus and Proconsul are ancestralapes.

III. Australopithecus is a human ancestor.

A. I only

B. I and II only

C. III only

D. I, II and III

46

GO ON TO THE NEXT PAGE.

5

10

15

20

25

30

35

40

45

50

55

60

65

70

Full-Length Test 5 06/27/2003 12:17 PM Page 46

Page 47: MCAT Full Length5

47

134. Based on the information presented in the passage,what would the author believe is most significant toanthropologists regarding the findings of molecularphylogenies?

A. It is less likely that Ramapithecus could havebeen a human ancestor.

B. Fossil evidence is fragmentary and subject tomore than one interpretation.

C. Ape and human origins prior to the ape-humansplit are much more closely related than waspreviously thought.

D. The invention of the molecular clock was madepossible.

135. Which of the following statements would be mostrelevant to the first paragraph of the passage?

A. Almost 200 years ago, French paleontologistCuvier boasted that he could reconstruct an ani-mal from its teeth alone. But paleontologistshave learned the hard way that teeth do not tellthe whole story.

B. Ramapithecus, first found in India, was namedin the 1930s after the Hindu prince “Rama,”hence “Rama’s ape.”

C. The famous nineteenth-century anatomistThomas Henry Huxley, in his 1863 book, Evi-dence As to Man’s Place in Nature, argued onanatomical grounds that humans were most sim-ilar to the African apes, a conclusion confirmed100 years later by biochemistry.

D. A comparison of developmental stages amongvertebrates led Ernst Haeckel (1834–1919) topropose his famous principle “ontogeny recapit-ulates phylogeny.”

136. Which of the following would most strengthen theclaim that molecular anthropology has practicalvalue?

A. Evidence that ape and human proteins haveevolved at the same rate

B. The finding by paleontologists that the Asianapes did not split off earlier than the rest

C. Evidence that Australopithecus was an earlyhuman

D. The finding of a complete intact fossil specimenof Ramapithecus with characteristics that are asmuch apelike as human

137. The author claims, but offers no supporting evidence,that:

A. the analysis of fossil bones and teeth is subjec-tive.

B. Ramapithecus is ancestral to the gorilla.

C. the molecular clock has statistical constancy.

D. we cannot look at human origins as an isolatedphenomenon.

STOP. IF YOU FINISH BEFORE TIME IS CALLED,CHECK YOUR WORK.YOU MAY GO BACK TO ANYQUESTION IN THIS SECTION ONLY.

Full-Length Test 5 06/27/2003 12:17 PM Page 47

Page 48: MCAT Full Length5

Full-Length Test 5 06/27/2003 12:17 PM Page 48

Page 49: MCAT Full Length5

Writing SampleTime: 60 Minutes

2 Prompts, Separately Timed:30 Minutes

DO NOT BEGIN THIS SECTION UNTIL YOU ARE TOLD TO DO SO.

Full-Length Test 5 06/27/2003 12:17 PM Page 49

Page 50: MCAT Full Length5

50

GO ON TO THE NEXT PAGE.GO ON TO THE NEXT PAGE.

WRITING SAMPLE

DIRECTIONS: This is a test of your writing skills. The test con-tains two parts. You will have 30 minutes to complete each part.

Your responses to the prompts given in this Writing Sample willbe written in the ANSWER DOCUMENT.Your response to Part 1must be written only on the answer sheets marked “1,” and yourresponse to Part 2 must be written only on the answer sheetsmarked “2.” You may work only on Part 1 during the first 30 min-utes of the test and only on Part 2 during the second 30 minutes.If you finish writing on Part 1 before the time is up, you mayreview your work on that part, but do not begin writing on Part 2.If you finish writing on Part 2 before the time is up, you mayreview your work only on part 2.

Use your time efficiently. Before you begin writing a response,read the assignment carefully and make sure you understandexactly what you are being asked to do. You may use the spacebelow each writing assignment to make notes in planning yourresponses.

Because this is a test of your writing skills, your response to eachpart should be an essay composed of complete sentences andparagraphs, as well organized and clearly written as you canmake it in the alloted time. You may make corrections or addi-tions neatly between the lines of your responses, but do not writein the margins of the answer booklet.

There are six pages in your answer booklet to write yourresponses, three pages for each part of the test. You are notrequired to use all of the pages, but to be sure that you haveenough space for each essay, do not skip lines.

Essays that are illegible cannot be scored.

Full-Length Test 5 06/27/2003 12:17 PM Page 50

Page 51: MCAT Full Length5

51

GO ON TO THE NEXT PAGE.GO ON TO THE NEXT PAGE.

Part 1

Consider this statement:

In the recording of history, it is impossible to be objective.

Write a unified essay in which you perform the following tasks. Explain what you think the above state-ment means. Describe a specific situation in which it might be possible to be objective in the recordingof history. Discuss what you think determines when objectivity in the recording of history is possible andwhen it is not.

Full-Length Test 5 06/27/2003 12:17 PM Page 51

Page 52: MCAT Full Length5

52

GO ON TO THE NEXT PAGE.GO ON TO THE NEXT PAGE.

Part 2

Consider this statement:

The best politician is the one most removed from politics.

Write a unified essay in which you perform the following tasks. Explain what you think the abovestatement means. Describe a specific situation in which the best politician might not be the one mostremoved from politics. Discuss what you think determines whether or not the best politician is the onemost removed from politics.

Full-Length Test 5 06/27/2003 12:17 PM Page 52

Page 53: MCAT Full Length5

Biological SciencesTime: 100 MinutesQuestions 138–214

DO NOT BEGIN THIS SECTION UNTIL YOU ARE TOLD TO DO SO.

Full-Length Test 5 06/27/2003 12:17 PM Page 53

Page 54: MCAT Full Length5

54

GO ON TO THE NEXT PAGE.GO ON TO THE NEXT PAGE.GO ON TO THE NEXT PAGE.

BIOLOGICAL SCIENCES

DIRECTIONS: Most of the questions in the BiologicalSciences test are organized into groups, with a descrip-tive passage preceding each group of questions. Studythe passage, then select the single best answer to eachquestion in the group. Some of the questions are notbased on a descriptive passage; you must also selectthe best answer to these questions. If you are unsure ofthe best answer, eliminate the choices that you knoware incorrect, then select an answer from the choicesthat remain. Indicate your selection by blackening thecorresponding circle on your answer sheet. A periodictable is provided below for your use with the questions.

1

H

1.0

2

He

4.0

3

Li

6.9

4

Be

9.0

5

B

10.8

6

C

12.0

7

N

14.0

8

O

16.0

9

F

19.0

10

Ne

20.2

11

Na

23.0

12

Mg

24.3

13

Al

27.0

14

Si

28.1

15

P

31.0

16

S

32.1

17

Cl

35.5

18

Ar

39.9

19

K

39.1

20

Ca

40.1

21

Sc

45.0

22

Ti

47.9

23

V

50.9

24

Cr

52.0

25

Mn

54.9

26

Fe

55.8

27

Co

58.9

28

Ni

58.7

29

Cu

63.5

30

Zn

65.4

31

Ga

69.7

32

Ge

72.6

33

As

74.9

34

Se

79.0

35

Br

79.9

36

Kr

83.8

37

Rb

85.5

38

Sr

87.6

39

Y

88.9

40

Zr

91.2

41

Nb

92.9

42

Mo

95.9

43

Tc

(98)

44

Ru

101.1

45

Rh

102.9

46

Pd

106.4

47

Ag

107.9

48

Cd

112.4

49

In

114.8

50

Sn

118.7

51

Sb

121.8

52

Te

127.6

53

I

126.9

54

Xe

131.3

55

Cs

132.9

56

Ba

137.3

57

La *

138.9

72

Hf

178.5

73

Ta

180.9

74

W

183.9

75

Re

186.2

76

Os

190.2

77

Ir

192.2

78

Pt

195.1

79

Au

197.0

80

Hg

200.6

81

Tl

204.4

82

Pb

207.2

83

Bi

209.0

84

Po

(209)

85

At

(210)

86

Rn

(222)

87

Fr

(223)

88

Ra

226.0

89

Ac †

227.0

104

Rf

(261)

105

Ha

(262)

106

Unh

(263)

107

Uns

(262)

108

Uno

(265)

109

Une

(267)

*

58

Ce

140.1

59

Pr

140.9

60

Nd

144.2

61

Pm

(145)

62

Sm

150.4

63

Eu

152.0

64

Gd

157.3

65

Tb

158.9

66

Dy

162.5

67

Ho

164.9

68

Er

167.3

69

Tm

168.9

70

Yb

173.0

71

Lu

175.0

90

Th

232.0

91

Pa

(231)

92

U

238.0

93

Np

(237)

94

Pu

(244)

95

Am

(243)

96

Cm

(247)

97

Bk

(247)

98

Cf

(251)

99

Es

(252)

100

Fm

(257)

101

Md

(258)

102

No

(259)

103

Lr

(260)

PERIODIC TABLE OF THE ELEMENTS

Full-Length Test 5 06/27/2003 12:17 PM Page 54

Page 55: MCAT Full Length5

Passage I (Questions 138–143)

Retinoblastoma (RB) is a cancer of the retinal cells thatdevelops in children between birth and the age of 4. RB iscaused by mutations in the RB gene, located on chromo-some 13. These mutations may be inherited from a parent(familial RB) or may arise spontaneously (sporadic RB).The wild-type RB gene is a tumor suppressor gene—agene that normally restrains cell growth. The wild-type RBgene product, the protein p105-RB, inhibits cell progres-sion from the G1 phase to the S phase of the cell cycle.Since RB develops when both copies of the wild-type geneare lost or inactivated, the RB gene is sometimes referredto as a recessive cancer gene.

In familial RB, a child is born with one inherited copyof the mutant RB gene, and one normal copy. The RB istriggered by a somatic mutation in a retinal cell that resultsin the loss of the normal RB gene. This acquired homozy-gosity causes the retinal cell to give rise to a tumor.

Sporadic RB is the result of two somatic mutations ina single retinal cell after birth. Loss of both normal copiesof the RB gene leads to the same tumor growth associatedwith familial RB. However, whereas familial RB patientsare usually blinded at an early age by multiple RB tumors,sporadic RB can develop at any age and typically producesonly one tumor.

Types of mutations that inactivate the RB gene includelarge deletions, small deletions affecting promoter sites,splicing errors that remove exons, and point mutations.

138. If R = the allele for the wild-type RB gene and r = theallele for the mutant RB gene, then what is the geno-type of an RB cancer cell in a child with familial RBand in a child with sporadic RB, respectively?

A. rr and rr

B. Rr and RR

C. Rr and Rr

D. rr and RR

139. What is the probability that a man with familial RBwill pass the mutant Rb gene on to a daughter?

A. 25%

B. 50%

C. 75%

D. 100%

140. According to the passage, p105-RB inhibits cell pro-gression from the G1 phase to the S phase of the cellcycle. This implies that p105-RB most likely:

A. inhibits microtubule formation, thus inhibitingcell movement.

B. inhibits DNA synthesis, thus inhibiting mitosis.

C. stimulates protein synthesis, thus promoting cellgrowth.

D. stimulates mutagenesis by binding to the RBgene, thus promoting RB development.

141. The protein products of tumor suppressor genes mayfunction in all of the following ways EXCEPT:

A. as an inhibitor of cell growth.

B. as an activator of programmed cell death.

C. as a regulator of cellular differentiation.

D. as an activator of cellular proliferation.

55

GO ON TO THE NEXT PAGE.GO ON TO THE NEXT PAGE.GO ON TO THE NEXT PAGE.

Full-Length Test 5 06/27/2003 12:17 PM Page 55

Page 56: MCAT Full Length5

142. Some retinal cells that are homozygous mutant at theRB gene loci still produce the p105-RB protein.Which of the following best explains how thisoccurs?

A. There is incomplete penetrance of the mutantRB gene.

B. The p105-RB produced by these retinal cells isnot functional.

C. The mutation in one of the RB genes is not in atranscription promoter site.

D. The retinal cells are phenotypically normal.

143. Some RB cancers are initiated by infection withSV40, a DNA virus. However, after infection bySV40, both copies of the RB gene in a retinal cell arestill normal and functional. Thus, the best explana-tion for the cancerous phenotype is that:

A. p105-RB cannot be synthesized after viralinfection.

B. SV40 inserts into an exon in the RB gene.

C. the paternal germline is more susceptible toviral infection.

D. an SV40 protein complexes with p105-RB,thereby inactivating it.

56

GO ON TO THE NEXT PAGE.GO ON TO THE NEXT PAGE.GO ON TO THE NEXT PAGE.

Full-Length Test 5 06/27/2003 12:17 PM Page 56

Page 57: MCAT Full Length5

Passage II (Questions 144–149)

In an attempt to develop a vaccine for pneumonia, FredGriffith performed a series of experiments in 1928 usingmice and two strains of the pneumococcus bacteria: a vir-ulent encapsulated strain and a nonvirulent unencapsulatedstrain. The encapsulated strain was called the “smoothstrain” because the colonies looked smooth on a cultureplate due to their polysaccharide capsules, whereas theunencapsulated strain was denoted as the “rough strain”due to the irregularity of its surface.

Four different groups of mice were injected with dif-ferent combinations of the bacterial strains. The experi-mental results are shown below in Table 1.

Table 1 Results of Griffith’s experiment

Mice Injected Bacteria Result

1 live smooth died

2 live rough survived

3 heat-killed smooth survived

4 heat-killed smooth + diedlive rough

Autopsies performed on Group 4 mice revealed bloodsamples filled with live smooth-strain bacteria. Somecomponent had been transferred from the heat-killedsmooth-strain bacteria to the live rough-strain bacteria,transforming them into the virulent smooth-strain bacteria.This process later became known as transformation. Grif-fith believed that protein from the dead smooth-strain bac-teria was the active transforming agent.

144. Which of the following experimental observationswould most contradict Griffith’s interpretation?

A. Live smooth-strain bacteria transformed rough-strain bacteria into smooth-strain bacteria.

B. The polysaccharide capsule from the smooth-strain bacteria did not transform rough-strainbacteria.

C. DNA from smooth-strain bacteria transformedrough-strain bacteria into smooth-strain bacteria.

D. The transforming agent in the Group 4 mice wascomprised mainly of amino acids.

145. Which of the following most accurately describeswhy the heat-killed smooth strain did NOT kill theGroup 3 mice?

A. The bacteria lacked polysaccharide capsules.

B. The bacteria’s DNA was denatured.

C. The bacteria could not cause infection.

D. The bacteria could not undergo cell division.

146. The Group 4 mice died because:

A. the rough strain transferred its messenger RNAto the heat-killed smooth strain.

B. the heat-killed smooth strain transferred itstransfer RNA to the rough strain.

C. the heat-killed smooth strain transferred itspolysaccharide capsules to the rough strain.

D. the heat-killed smooth strain transferred itsDNA to the rough strain.

57

GO ON TO THE NEXT PAGE.GO ON TO THE NEXT PAGE.GO ON TO THE NEXT PAGE.

Full-Length Test 5 06/27/2003 12:17 PM Page 57

Page 58: MCAT Full Length5

147. A colony of smooth strain bacteria is grown on a cul-ture containing an experimental drug that cleavesnucleic acid base sequences wherever adenine ispaired with uracil. Which of the following processeswill be directly affected?

I. Transcription

II. Translation

III. Transformation

A. I only

B. I and II only

C. I and III only

D. I, II, and III

148. Which of the following best describes the shape ofthe pneumococcus bacteria?

A. Rod-shaped

B. Spiral

C. Spherical

D. Elliptical

149. If Griffith had injected a fifth group of mice with acombination of heat-killed rough strain and heat-killed smooth strain, would the mice have died?

A. No, because the heat-killed rough strain cannotinfect mice.

B. No, because the heat-killed smooth strain can-not transform mice cells.

C. Yes, because the heat-killed rough strain istransformed into the smooth strain.

D. Yes, because the mice do not have acquiredimmunity to pneumococcus bacteria.

58

GO ON TO THE NEXT PAGE.GO ON TO THE NEXT PAGE.GO ON TO THE NEXT PAGE.

Full-Length Test 5 06/27/2003 12:17 PM Page 58

Page 59: MCAT Full Length5

Passage III (Questions 150–154)

Carboxylic acids (general formula = RCOOH) are veryuseful organic compounds, since they can be convertedinto a wide range of derivatives. In addition, they can besynthesized from a number of precursors. The most com-mon reactions are summarized in Figure 1.

(Note: R = alkyl or H; Compound V = carboxylic acid.)

Figure 1 Common reactions of carboxylic acid and itsderivatives

To investigate these reactions, a student carried out thefollowing procedure:

Compound A (C7H6O) was reacted with silver oxide(Ag2O) to form Compound B (C7H6O2). Compound Bwas found to be insoluble in water; however, upon theaddition of sodium hydroxide, Compound B dissolved inthe aqueous layer. A mixture of Compound B, CompoundC (CH4O), and mineral acid was then heated to reflux,resulting in the formation of Compound D (C8H8O2). The1H NMR spectrum of Compound D is shown in Figure 2.

Figure 2 1H NMR Spectrum of Compound D

150. What is the correct IUPAC name for Compound D?

A. Cyclohexanecarboxylic acid

B. 2-Methylbenzoic acid

C. Methyl benzoate

D. 3-Methoxybenzaldehyde

151. Which of the following is the correct structure ofCompound B?

A.

B.

C.

D.

H3C C C

O

OH

C

O

OH

C

O

OH

OOH

CH3

CH2 CH2

10 9 8 7 6 5

δ (ppm)

4 3 2 1 0

23 3

RCH2OH

R'OH, H+/∆R'OH

SOCl2

R CR X

R MgX

Mg/Ether

O

H

R C

O

O–Na+

R C

O

O R'

R C

O

OHR C

O

Cl

I IIIII

IV

VI

VII

VVIII

1) KMnO4OH–

2) H+

1) Ag2O, OH–

2) H+

1) CO2

2) H+

59

GO ON TO THE NEXT PAGE.GO ON TO THE NEXT PAGE.GO ON TO THE NEXT PAGE.

Full-Length Test 5 06/27/2003 12:17 PM Page 59

Page 60: MCAT Full Length5

152. Why is an acid catalyst used in the conversion ofCompound V to Compound VII?

A. The catalyst donates proton to the carbonyl oxy-gen which makes the carbonyl carbon more sus-ceptible to nucleophilic attack.

B. The catalyst donates a proton to the hydroxylgroup which makes the carbonyl carbon moresusceptible to nucleophilic attack.

C. The catalyst donates hydroxide ions whichattack the carbonyl carbon.

D. The catalyst accepts hydroxide ions which drivethe reaction to completion.

153. Which of the following processes will NOT result inthe formation of butanoic acid?

A. The reaction of butanol with potassium perman-ganate followed by acidification.

B. The reaction of butylmagnesium chloride withcarbon dioxide followed by acidification.

C. The reaction of sodium butanoate with aqueousmineral acid.

D. The reaction of propylmagnesium chloride withcarbon dioxide followed by acidification.

154. Would you expect Compound V to have a higherboiling point than Compound I? (Assume R = H)

A. No, because Compound I will undergointramolecular hydrogen bonding to a greaterdegree.

B. No, because Compound I will undergo inter-molecular hydrogen bonding to a greaterdegree.

C. Yes, because Compound V will undergointramolecular hydrogen bonding to a greaterdegree.

D. Yes, because Compound V will undergo inter-molecular hydrogen bonding to a greaterdegree.

60

GO ON TO THE NEXT PAGE.GO ON TO THE NEXT PAGE.GO ON TO THE NEXT PAGE.

Full-Length Test 5 06/27/2003 12:17 PM Page 60

Page 61: MCAT Full Length5

Passage IV (Questions 155–159)

Multiple sclerosis (MS) is a demyelinating diseaseafflicting oligodendrocytes, the myelin-producing cells ofthe central nervous system (CNS). MS is characterized byareas of demyelination, known as plaques, in both grayand white brain matter. These plaques are characterized bycenters with marked reduction in oligodendrocyte densityand borders that contain high concentrations of lympho-cytes. The demyelinated axons passing through theplaques are largely preserved.

Two theories have been advanced to explain the onsetand progression of the demyelination associated with MS.

Hypothesis I

Oligodendrocytes are damaged, rendering them inca-pable of manufacturing and maintaining myelin sheaths.Scavenger cells (e.g., macrophages) take up myelinreleased from the damaged regions and present it to circu-lating T cells and B cells as a novel antigen. Once theimmune cells recognize myelin as a foreign substance,they begin destroying oligodendrocytes in other regions ofthe CNS.

Hypothesis 2

Lymphocytes found in the tissue surrounding thebrain’s blood vessels initiate myelin degeneration throughthe release of lymphokines or immunoglobulins. Thisrelease may be initiated locally or as a result of an unre-lated process elsewhere in the body, such as an organ fight-ing a viral infection. Since the myelin loss is generated byexternal mediators, the outer layers of myelin aredestroyed first, leaving the oligodendrocytes vulnerable toattack by other cells scavenging for cellular debris.

155. A pathologist determined at autopsy that an MSpatient had several large plaques in the cerebellum.When the patient was alive, all of the following func-tions were most likely intact EXCEPT:

A. visual image processing.

B. speech.

C. fine motor coordination

D. respiration.

156. In order to determine whether Hypothesis 1 is thedominant mechanism for the initiation and progres-sion of MS, a scientist performs a series of studies. Ifit is shown that the immune system is the primarysystem involved, it can be concluded that:

A. Hypothesis 1 is true and Hypothesis 2 is false.

B. Hypothesis 2 is true and Hypothesis 1 is false.

C. both Hypothesis 1 and Hypothesis 2 are false.

D. neither Hypothesis 1 nor Hypothesis 2 can beeliminated.

157. Factors that would help explain findings of reducednervous system function in MS patients wouldinclude all of the following EXCEPT:

A. loss of insulating properties of myelin.

B. decreased ability of damaged nerve axons toregenerate.

C. diminished axon conduction velocity.

D. diffuse central nervous system demyelination.

158. Based on information in the passage, which of thefollowing new developments would most WEAKENHypothesis 2?

A. The interface between the axon and the myelinsheath is the site of initial damage.

B. The majority of MS patients contracted themeasles virus 15 to 20 years before the onset ofMS symptoms.

C. T cells and B cells release lymphocyte stimulat-ing substances when macrophages present themwith myelin as a novel antigen.

D. Lymphocytes can be induced to release lym-phokines by chemical stimulation.

159. Assuming a patient has MS plaques limited to thebrain, which of the following functions is most likelyNOT affected?

A. Eye muscle control

B. Reasoning ability

C. Sense of touch in arms

D. Knee jerk reflex

61

GO ON TO THE NEXT PAGE.GO ON TO THE NEXT PAGE.GO ON TO THE NEXT PAGE.

Full-Length Test 5 06/27/2003 12:17 PM Page 61

Page 62: MCAT Full Length5

Questions 160 through 164 are NOT basedon a descriptive passage.

160. The following molecules are examples of:

A. cis/trans isomers.

B. enantiomers.

C. diastereomers.

D. meso compounds.

161. Which of the following does NOT explain how it ispossible for a fetus to obtain enough oxygen whenblood entering the fetus has a PO2

of only 20 mmHg?

A. Fetal hemoglobin has a greater affinity for oxy-gen than does maternal hemoglobin.

B. At low PO2fetal hemoglobin can carry 20–30%

more oxygen than can maternal hemoglobin.

C. The concentration of hemoglobin in fetal bloodis 50% higher than that of maternal blood.

D. Fetal lungs are not functional prior to birth.

162. The reactivity towards SN1 reactions decreases fromleft to right in the following compounds:

Which of the following best accounts for thisobservation?

A. The steric hindrance in the compoundsincreases from left to right.

B. The reactivity of the compounds decreases fromleft to right.

C. The ability of the compounds to form a planarcarbocation decreases from left to right.

D. The ability of the compounds to undergo back-side attack by the nucleophile decreases fromleft to right.

163. Compared to an E. coli living within the humanintestinal tract, a paramecium would produce howmuch ATP from the degradation of one molecule ofglucose?

A. the same

B. 1/18 as much

C. 18 times as much

D. 36 times as much

164. Due to the numerous similarities between modernmitochondria and bacteria, it is believed that present-day eukaryotic cells are the evolutionary product of aunique relationship between primitive aerobicprokaryotes and primitive anaerobic eukaryotes. Thistheory is referred to as:

A. the chemiosmotic hypothesis.

B. the endosymbiotic hypothesis.

C. the coevolutionary hypothesis.

D. the endocytotic hypothesis.

Br Br Br

I II IIICH3

CH3 CH3 CH3

62

GO ON TO THE NEXT PAGE.GO ON TO THE NEXT PAGE.GO ON TO THE NEXT PAGE.

Full-Length Test 5 06/27/2003 12:18 PM Page 62

Page 63: MCAT Full Length5

Passage V (Questions 165–169)

Cortisol is a glucocorticoid steroid hormone producedby the adrenal cortex. The primary function of cortisol isto stimulate the formation of glucose from the body’sstores of protein and fat during periods of stress. Cortisolsynthesis and secretion are tightly controlled in a feedbackloop with the hypothalamus and the pituitary gland. Neu-rons in the hypothalamus release a peptide called corti-cotropin-releasing hormone (CRH), which causes thepituitary gland to release adrenocorticotropic hormone(ACTH). ACTH circulates in the bloodstream and stimu-lates the adrenals to produce cortisol. A high plasma corti-sol concentration, in turn, acts on the hypothalamus todecrease CRH and on the pituitary to suppress ACTH.

Cushing’s syndrome is defined as an excess concentra-tion of cortisol in the bloodstream. Symptoms includehypertension, truncal obesity, diabetes, reproductiveabnormalities, acne, and muscle weakness. Cushing’s syn-drome is most commonly the result of overproduction ofACTH by the pituitary. In this case, the pituitary respondsto negative feedback only at abnormally high levels of cor-tisol. Cushing’s syndrome can also be caused by adrenaltumors, by ectopic ACTH-producing tumors (such as oatcell carcinomas of the lungs), and by the administration ofexcessive amounts of exogenous steroids.

165. The muscle weakness exhibited by patients withCushing’s syndrome is most likely due to the:

A. abnormally high concentration of glucose in theblood.

B. decreased ability to respond to stressful situations.

C. fluctuations in CRH and ACTH concentrations.

D. increased protein catabolism.

166. Which of the following would most likely beobserved in a patient with Cushing’s syndromecaused by an adrenal tumor?

A. Low CRH, low ACTH, high cortisol

B. Low CRH, high ACTH, high cortisol

C. High CRH, high ACTH, high cortisol

D. High CRH, low ACTH, high cortisol

167. Cortisol decreases a cell’s utilization of glucose byinhibiting glucose transport into the cell. As a result ofthis, a patient with Cushing’s syndrome will have a(n):

A. increased sensitivity to insulin.

B. decreased sensitivity to insulin.

C. decreased recovery from low plasma glucoselevels.

D. decreased plasma glucose levels.

168. Which of the following accounts for the masculinesecondary sex characteristics sometimes seen inwomen with Cushing’s syndrome?

A. Cortisol inhibits estrogen secretion.

B. Increased ACTH stimulates adrenal androgenproduction.

C. ACTH stimulates the testes to produce testos-terone.

D. ACTH converts cortisol to testosterone.

169. Dexamethasone, a synthetic glucocorticoid withgreater suppressive activity than cortisol, is used as adiagnostic tool to determine the cause of Cushing’ssyndrome in patients. If a patient is given a high doseof dexamethasone and cortisol concentration isobserved to decrease, the syndrome is most likely theresult of:

A. pituitary-dependent oversecretion of ACTH.

B. insufficient secretion of CRH.

C. adrenal-dependent oversecretion of cortisol.

D. over-ingestion of exogenous steroids.

63

GO ON TO THE NEXT PAGE.GO ON TO THE NEXT PAGE.GO ON TO THE NEXT PAGE.

Full-Length Test 5 06/27/2003 12:18 PM Page 63

Page 64: MCAT Full Length5

Passage VI (Questions 170–174)

Two of the most common carbonyl reducing agents inuse today are lithium aluminum hydride (LiAlH4) andsodium borohydride (NaBH4).

These reducing agents transfer a hydride ion from thereducing agent to a positively-polarized carbon. Theresulting alkoxide ion then picks up a proton to form theproduct (Figure 1).

Figure 1 Mechanism for the reduction of carbonyl com-pounds

Although both reagents are believed to react throughthe same mechanism, lithium aluminum hydride is lessselective than sodium borohydride (i.e., lithium aluminumhydride will indiscriminately reduce any electrophilically-polarized carbon, whereas sodium borohydride will onlyreduce certain electrophilically-polarized carbons). In thereduction of conjugated carbon-carbon bonds, however,the opposite trend occurs: Lithium aluminum hydride isthe more selective reducing agent. This trend is evident inthe conversion of cinnamaldehyde to cinnamyl alcohol(Reaction 1 and Reaction 2).

Another difference between LiAlH4 and NaBH4 is intheir handling requirements. Lithium aluminum hydridereacts violently with water, so it must be dissolved in anethereal solvent such as THF. On the other hand, sincesodium borohydride is less reactive than lithium aluminumhydride, it can be dissolved in an aqueous solvent.

Reaction 1

Reaction 2

CH CHCH

Cinnamaldehyde

Cinnamyl Alcohol

1. NaBH4' MeOH

2. H+, H2O

O

CH CHCH2OH

3-Phenyl-1-Propanol

CH2CH2CH2OH

+

CH CHCH

Cinnamaldehyde

Cinnamyl Alcohol

1. LiAlH4' THF

2. H+, H2O

O

CH CHCH2OH

H3M H

R

R

R

R

H+

C O O C O

R

R

H C OH

Alkoxide ion

Alcohol

Aldehyde or Ketone

(Note: M = B or A)

64

GO ON TO THE NEXT PAGE.GO ON TO THE NEXT PAGE.GO ON TO THE NEXT PAGE.

Full-Length Test 5 06/27/2003 12:18 PM Page 64

Page 65: MCAT Full Length5

170. What is the major product of the reaction shownbelow?

171. When 1-deuteropropanal (CH3CH2CDO) is reactedwith LiAlH4 followed by the addition of water, 1-deutero-1-propanol (CH3CH3CHDOH) is formed.The product is most likely:

A. an R enantiomer.

B. an S enantiomer.

C. achiral.

D. a racemic mixture.

172. A chemist wished to carry out the conversion of 9-fluorenone to 9-hydroxyfluorene, shown below.

Which of the following reagents could be used tocarry out this conversion?

I. LiAlH4II. Cl2/FeCl3

III. NaBH4

A. I only

B. II only

C. I and III only

D. I, II and III

173. Which of the following compounds will give thehighest yield of saturated product upon reaction withsodium borohydride?

174. Which of the following reagents could NOT be usedto convert cis-2-butenal to 1-butanol?

A. NaBH4B. H2/Pd

C. H2/Ni

D. H+/H2O

Ph

A. C.

B. D.

C C

O

Ph

CHO

CHOCHO

H

O OH

9-Fluorenone 9-Hydroxyfluorene

1. LiAlH4'

2. H+, H2O

OC2H5

Note ∅ =

O

OC2H5

O

OC2H5

OC2H5

HO

H

OH

O

A. C.

B. D.

65

GO ON TO THE NEXT PAGE.GO ON TO THE NEXT PAGE.GO ON TO THE NEXT PAGE.

Full-Length Test 5 06/27/2003 12:18 PM Page 65

Page 66: MCAT Full Length5

Passage VII (Questions 175–181)

Human bone marrow contains cells called pluripotenthemopoietic stem cells. These cells are capable of differ-entiating into red blood cells, white blood cells, or plateletsdepending on the presence of specific growth and differ-entiation inducers. Production of these inducers is in turncontrolled by variables outside the bone marrow.

The primary factor controlling the production of redblood cells (RBCs) is the glycoprotein hormone erythro-poietin. Erythropoietin is produced primarily by the kidneyand is released in response to a deficiency in oxygen deliv-ery to the tissues.

Anemia is defined as a deficiency in the number ofRBCs. Anemia can result from one of two primary mech-anisms: insufficient RBC production (e.g., iron deficiency)or accelerated RBC loss (e.g., hemorrhaging). In bothtypes of anemia, the concentration of hemoglobin and thehematocrit (the percentage of blood constituted by RBCs,normally 40–45%) are below normal. One way to differ-entiate whether an anemia results from underproduction oroverdestruction of RBCs is to take a reticulocyte count.Reticulocytes are immature RBCs. There will be anincrease in the number of reticulocytes in the blood ofpatients suffering from increased RBC loss.

175. Which of the following experimental findings mostsupports the conclusion that erythropoietin produc-tion is stimulated by decreased oxygen delivery andnot simply decreased RBC number?

A. Erythropoietin is secreted in response to movingto a high altitude.

B. Erythropoietin is secreted in patients with irondeficiency anemia.

C. Erythropoietin is secreted in response to hemor-rhaging.

D. Erythropoietin is secreted in patients with ane-mia due to a Vitamin B12 deficiency.

176. Aplastic anemia occurs when the bone marrow is notfunctioning. A bone marrow sample will show ahypocellular marrow. Which of the following wouldyou expect to see in a patient with aplastic anemia?

A. An increased number of circulating reticulo-cytes

B. Iron deficiency

C. An increased number of platelets and whiteblood cells, but a decreased number of RBCs

D. A decreased number of platelets, white bloodcells, and RBCs

177. In sickle cell anemia, the presence of abnormalhemoglobin makes these cells very susceptible tobreakage and destruction upon exposure to oxygen.Which of the following would you expect in patientswith sickle cell anemia?

I. A decreased hematocrit

II. An increased reticulocyte count

III. White blood cell abnormalities

A. I only

B. I and II only

C. II and III only

D. I, II and III

178. In an anemic patient, you would expect to see all ofthe following EXCEPT:

A. increased concentration of erythropoietin.

B. decreased blood viscosity.

C. increased oxygen-carrying capacity.

D. decreased hemoglobin concentration.

66

GO ON TO THE NEXT PAGE.GO ON TO THE NEXT PAGE.GO ON TO THE NEXT PAGE.

Full-Length Test 5 06/27/2003 12:18 PM Page 66

Page 67: MCAT Full Length5

179. Hereditary spherocytosis is an autosomal dominantdisease resulting from an abnormality in the RBC’scytoskeleton. What is the probability that a childborn to a man with hereditary spherocytosis and anormal woman will develop the disease? (Note: thechild’s paternal grandmother was unaffected.)

A. 0%

B. 25%

C. 50%

D. 100%

180. Where would you expect to find erythopoietinreceptors?

A. Inside RBCs

B. On the cell membranes of RBCs

C. Inside stem cells of the bone marrow

D. On the cell membranes of stem cells in the bonemarrow

181. All of the following are true of RBCs EXCEPT:

A. They cannot synthesize ATP.

B. They are incapable of cell division.

C. They do not have mitochondria.

D. They are derived from the same original type ofcells as white blood cells.

67

GO ON TO THE NEXT PAGE.GO ON TO THE NEXT PAGE.GO ON TO THE NEXT PAGE.

Full-Length Test 5 06/27/2003 12:18 PM Page 67

Page 68: MCAT Full Length5

Passage VIII (Questions 182–186)

Receptors for the psychoactive drug delta-9-tetrahy-drocannabinol (THC) have been found in the human brainand white blood cells, and more recently, in rat spleen andlymph nodes. Similar receptors have also been found incertain marine organisms, leading some researchers to sug-gest that THC is part of an ancient and widespread cellularsignaling system.

After confirming gross binding of THC to neuron andwhite blood cell (WBC) receptor systems in whole cellcultures, a researcher wants to compare the receptor sitesthemselves. He uses PCR technology to determine theDNA sequence for both the WBC and neuronal THCreceptors. The researcher inserts the genetic codes forthese receptors into cells grown in the lab. Three sets ofcell cultures are prepared: one with the neuronal receptor,one with the WBC receptor, and a control. After 2 days ofgrowth, all of the cultures are exposed to THC to deter-mine binding affinity. The results are shown below.

Table 1 THC binding affinity to specific receptor sites

WBC Neuronalreceptors receptors Control

THC 0.44 1.01 0.03

binding 0.45 0.99 0.07

affinity 0.39 1.00 0.02

Two theories are proposed to explain this difference inreceptor binding.

Theory A

Neuronal THC receptors and WBC THC receptorshave different affinities for THC.

Theory B

The WBC cell membrane has a membrane protein thatis in proximity to the THC receptor and interferes withTHC binding.

182. If Theory B is correct, which of the following wouldbest account for why THC binds with less affinity tothe WBC receptor than to the neuronal receptor?

A. Competitive inhibition at the WBC receptor site

B. Noncompetitive inhibition at the WBC receptorsite

C. WBC receptors and neuronal receptors differ intheir attraction for THC

D. WBCs have fewer membrane proteins

183. If Theory A is correct, differences in receptor affinitycould best be explained by which of the following?

A. There are more neurons than lymphocytes in thebody.

B. Neurons are larger than lymphocytes.

C. Second messenger cascade reactions amplifythe effect of THC binding in neurons to a muchgreater degree than in WBCs.

D. The cannabinol molecule fits into the neuronalTHC receptor site more easily than into theWBC THC receptor site.

184. THC is found to induce synthesis of a neuropolypep-tide called ASF (Appetite Stimulating Factor). Inorder to increase serum ASF levels, THC’s site ofaction within the cell would most likely be:

A. the nucleus.

B. mitochondria.

C. lysosomes.

D. cilia.

68

GO ON TO THE NEXT PAGE.GO ON TO THE NEXT PAGE.GO ON TO THE NEXT PAGE.

Full-Length Test 5 06/27/2003 12:18 PM Page 68

Page 69: MCAT Full Length5

185. Which of the following findings would favor TheoryA over Theory B?

A. T4 cells, which are a type of WBC, bind THCwith greater affinity than all other WBCs.

B. Radioactively labeled THC concentrates morethan 2:1 in neurons versus WBCs.

C. There are significant structural differencesbetween the neuronal THC receptor and theWBC THC receptor.

D. THC has enhanced potency in severely WBC-depleted AIDS patients.

186. If Theory A is correct, which of the following canmost likely be concluded?

A. The DNA sequence for the WBC THC receptoris different than the DNA sequence for the neu-ronal THC receptor.

B. The DNA sequence for the WBC THC receptoris identical to the DNA sequence for the neu-ronal THC receptor.

C. A cell poison that inhibits ATP productionwould not affect the experimental results.

D. The control cell cultures had a greater concen-tration of WBC THC receptors than neuronalTHC receptors.

69

GO ON TO THE NEXT PAGE.GO ON TO THE NEXT PAGE.GO ON TO THE NEXT PAGE.

Full-Length Test 5 06/27/2003 12:18 PM Page 69

Page 70: MCAT Full Length5

Questions 187 through 191 are NOT basedon a descriptive passage.

187. Which of the following animals most likely has theLEAST dense bones?

A. Iguana

B. Squirrel

C. Whale

D. Sparrow

188. Oral contraceptives contain a combination of syn-thetic estrogens and progestins. By which of the fol-lowing methods do oral contraceptives suppressovulation?

A. The presence of high levels of estrogen and pro-gesterone promote secretion of gonadotropin-releasing hormones, thereby inhibitingluteinizing hormone and follicle stimulatinghormone.

B. The presence of high levels of estrogen and pro-gesterone inhibit secretion of gonadotropin-releasing hormones, thereby inhibitingluteinizing hormone and follicle stimulatinghormone.

C. The presence of high levels of estrogen and pro-gesterone induce menstruation to occur.

D. Estrogen and progesterone stimulate the pro-duction of human chorionic gonadotropin(HCG), which prevents ovulation from occur-ring.

189. What is the major product of the following reaction?

A.

C CH CH3

CH3

CH3

OH

CH3H+

heat

C CH CH2

CH3

CH3

CH3

C.

C C CH3

CH3

CH3

O

CH3

D.

C CH CH3

CH3

OH

CH2

B.CH3

C C

CH3

CH3

CH3

70

GO ON TO THE NEXT PAGE.GO ON TO THE NEXT PAGE.GO ON TO THE NEXT PAGE.

Full-Length Test 5 06/27/2003 12:18 PM Page 70

Page 71: MCAT Full Length5

190. The reaction below is an example of:

A. electrophilic addition.

B. nucleophilic substitution.

C. hemiacetal formation.

D. esterification.

191. The role played by tropomyosin in skeletal musclecontraction is most similar to that of:

A. an inducer.

B. an immunoglobulin.

C. a repressor.

D. a catalyst.

OCH2O2CCH3

O2CCH3CH3CO2

CH3CO2CH3CO2

CH2OH

(CH3CO)2O

PyridineOHOH

HOHO

O

71

GO ON TO THE NEXT PAGE.GO ON TO THE NEXT PAGE.GO ON TO THE NEXT PAGE.

Full-Length Test 5 06/27/2003 12:18 PM Page 71

Page 72: MCAT Full Length5

Passage IX (Questions 192–198)

The human body responds to starvation by using dif-ferent substances for its primary metabolic fuel. During thefirst day of starvation, low blood glucose inhibits insulinsecretion and stimulates glucagon secretion. Consequently,glycolysis is inhibited and glycogen utilization and glu-cose synthesis are promoted. In adipose cells, triacylglyc-erols are hydrolyzed, leading to the release and oxidationof fatty acids. In muscle, protein is degraded to providesome of the precursors for glucose synthesis. Glycerolderived from triacylglycerol cleavage is another raw mate-rial used for glucose synthesis. The use of fatty acids byliver and muscle tissue allows glucose to be conserved foruse by the brain.

After three days of starvation, the liver produces massquantities of ketone bodies (acetoacetate and 3-hydroxy-butyrate) from acetyl CoA generated from fatty acid oxi-dation. The brain begins to consume acetoacetate in placeof glucose, thereby lowering the need for blood glucoseeven further. After several weeks of starvation, ketonebodies become the major fuel of the brain, markedlydiminishing the need for glucose. Hence, less protein (andtherefore less muscle tissue) is degraded than in the earlierstages of starvation.

Migratory birds’ fuel storage system is another exam-ple of the biological value of triacylglycerols, which holdsix times the energy of glycogen. The birds have large fatdeposits under their skin, as well as in their abdomens,muscle and livers. These deposits are so quickly and effi-ciently mobilized that long migratory flights are possiblewith little protein degradation.

192. During periods of starvation, the human body cangenerate glucose from:

I. glycogen.

II. the glycerol portion of triacylglycerol.

III. the fatty acid portion of triacylglycerol.

IV. amino acids.

A. I and IV only

B. I, II and III only

C. I, II and IV only

D. I, II, III, and IV

193. It can be inferred form the passage that glucagon:

A. increases the concentration of glucose in theblood.

B. increases the concentration of triacylglycerol inthe blood.

C. decreases the concentration of glucose in theblood.

D. decreases the concentration of insulin in theblood.

194. Glycogen is localized mainly in muscle and liver tis-sue. If it were possible for a migratory bird to useglycogen instead of triacylglycerol as its main storedfuel for a long flight, which of the following wouldmost likely occur?

A. The bird would use glucose as its mainmetabolic fuel during migratory flight.

B. The bird would not be able to migrate, becauseit would have to carry too much weight in fuelfor the flight.

C. The bird’s brain cells would use acetoacetate astheir primary fuel during migratory flight.

D. The bird would not be able to complete themigratory flight because it would lack a readysupply of glucose.

195. Which of the following is LEAST likely to occurduring starvation?

A. Gluconeogenesis is promoted.

B. Insulin secretion is inhibited.

C. Triacylglycerol is hydrolyzed.

D. Protein is synthesized.

72

GO ON TO THE NEXT PAGE.GO ON TO THE NEXT PAGE.GO ON TO THE NEXT PAGE.

Full-Length Test 5 06/27/2003 12:18 PM Page 72

Page 73: MCAT Full Length5

196. Which of the following best explains why, after sev-eral weeks of starvation, acetyl CoA is used almostexclusively to produce ketone bodies?

A. Gluconeogenesis depletes the supply ofoxaloacetate, which is essential for the entry ofacetyl CoA into the Krebs cycle.

B. Starvation leads to the selective production ofthe enzyme that catalyzes the formation ofketone bodies.

C. Acetyl CoA cannot be converted into pyruvate;therefore, it cannot enter the gluconeogenicpathway.

D. Triacylglycerols are mobilized from adipose tis-sue.

197. Why does the brain use glucose rather than fattyacids as its major metabolic fuel during the earlystages of starvation?

A. Liver and muscle cells selectively utilize fattyacids at the expense of the brain’s metabolicrequirements.

B. The concentration of triacylglycerol in theblood is not high enough to provide for thebrain’s metabolic requirements.

C. Glucose can cross the blood-brain barrier viaspecific carrier molecules, while fatty acids can-not.

D. Fatty acids are more easily oxidized in periph-eral tissues than in the central nervous system.

198. Which of the following is most likely to occur imme-diately following a meal?

A. β-oxidation of fatty acids

B. Inhibition of glucagon secretion

C. Increased cellular fermentation

D. Gluconeogenesis

73

GO ON TO THE NEXT PAGE.GO ON TO THE NEXT PAGE.GO ON TO THE NEXT PAGE.

Full-Length Test 5 06/27/2003 12:18 PM Page 73

Page 74: MCAT Full Length5

Passage X (Questions 199–203)

Bimolecular nucleophilic substitution (SN2) reactionsplay an important role in a number of organic syntheses.

SN2 reactions are most successful when carried out inpolar aprotic solvents rather than polar protic solvents.However, since polar aprotic solvents often possess highboiling points, they are difficult to remove at the end of thereaction. Nonpolar aprotic solvents such as hydrocarbonspossess low boiling points and so would seem a favorablealternative to polar aprotic solvents; however, they do notdissolve ionic compounds—at least, not until the introduc-tion of phase-transfer catalysts.

Phase-transfer catalysis (Figure 1) involves the use oftwo immiscible phases—an organic phase and an aqueousphase. The addition of a phase-transfer catalyst (Q+X–)ensures that the nucleophile (Nu–) dissolves in the low-boiling organic phase in the form of an ion pair (Q+Nu–).The nucleophile dissolves because the cation provided bythe catalyst, Q+, is lipophilic—it prefers the organic phaseto the aqueous phase. As a result, the nucleophile can reactwith the substrate (RX) to form the SN2 product (RNu),and can be easily removed from the solvent while theoriginal phase-transfer catalyst is transported back to theaqueous layer.

Figure 1 Phase-transfer catalysis

Phase-transfer catalysts are usually quaternary ammo-nium salts. An example of the use of this catalyst is theconversion of 1-chlorooctane to nonanenitrile (Reaction 1).

R4N+Br–

CH3(CH2)7Cl (in decane) CH3(CH2)7CNNaCN (aq)

105°C1-Chlorooctane Nonanenitrile

Reaction 1

The use of phase-transfer catalysts is not restricted tonucleophilic substitution reactions; they can also be used inoxidation and elimination reactions:

Phase –C6H5CH2OH (in benzene) C6H5COOH+ KMnO4(aq) transfer

catalysis

Reaction 2

Phase –C6H5CH2CH2Br (in CH2Cl2) C6H5CHCH2

+ NaOH(aq) transfercatalysis

Reaction 3

199. What is the correct IUPAC name of the reactant inReaction 3?

A. (2-Bromoethyl) benzene

B. 1-Bromo-2-ethylbenzene

C. 1-Bromo-2-benzylethane

D. 2-Bromo-1-ethylbenzene

200. Which of the following is NOT a quaternary ammo-nium salt?

A. (CH3)4N+Br–

B. C6H5CH2N+(CH2CH3)3Cl–

C. CH3NH3+Cl–

D. [CH3(CH2)6CH2]3N+CH3Cl–

Q+ X-

Q+ X-

X-

Q+Nu-

Q+Nu-

RNu RX

Nu-Aqueous

Phase

OrganicPhase

+

+ +

+

74

GO ON TO THE NEXT PAGE.GO ON TO THE NEXT PAGE.GO ON TO THE NEXT PAGE.

Full-Length Test 5 06/27/2003 12:18 PM Page 74

Page 75: MCAT Full Length5

201. Which of the following represents the transition stateformed in Reaction 1?

202. If aqueous sodium bicarbonate and the organic layerobtained in Reaction 2 are vigorously shakentogether in a separatory funnel, benzoic acid:

A. reacts to form a sodium salt, but remains in theorganic layer.

B. reacts to form a sodium salt and moves to theaqueous layer.

C. doesn't form a sodium salt and remains in theorganic layer.

D. doesn't form a sodium salt, but moves to theaqueous layer.

203. Which of the following solvents would be mosteffective in an SN2 reaction?

C CH3

H

CH3

CH3

CH3C

CH2OHHOCH2

O

O O

S

OH

A.

B.

C.

D.

CN C

H

H Cl

(CH2)6CH3

δ−

(CH2)6CH3

CN C

H

H

Clδ−

A.

B.

(CH2)6CH3

CN C

H

H

Clδ− δ−

C.

(CH2)6CH3CN C

H

H Cl

δ−

D.

δ +

δ +

δ +

75

GO ON TO THE NEXT PAGE.GO ON TO THE NEXT PAGE.GO ON TO THE NEXT PAGE.

Full-Length Test 5 06/27/2003 12:18 PM Page 75

Page 76: MCAT Full Length5

Passage XI (Questions 204–209)

Isomers are defined as compounds with the samemolecular formula but different structures. The figurebelow shows the isomer of three different compounds.

When compound C is treated with hydrogen bromidefollowed by heating in the presence of potassium hydrox-ide in ethanol, compound D is formed (Reaction 1).

Reaction 1

204. The formation of Compound D from Compound Coccurs by:

A. nucleophilic substitution.

B. free-radical addition.

C. electrophilic addition followed by elimination.

D. nucleophilic addition followed by elimination.

205. Compound B exists in equilibrium with the follow-ing isomer:

Which isomer will be favored in the equilibrium?

A. Compound B, because this molecule has lessnonbonded strain.

B. Compound B, because this molecule has lessangle strain.

C. Compound E, because this molecule has lessnonbonded strain.

D. Compound E, because this molecule has lessangle strain.

206. Compound C and Compound D are classed as:

A. cis/trans isomers.

B. configurational isomers.

C. meso compounds.

D. enantiomers.

CH3H

H

H

Compound E

H

HH

HH

H

H

Cl

CH3CH2 CH2CH3

CH2CH3CH3CH2

HBr

Br

H H H H

H

∆ KOH/EthanolCompound C

CH3CH2

CH2CH3H

H

Compound D

Cl Cl

CH3

CH3CH2 CH2CH3

CH3

CH3Br

BrH

H

H

H H

I

Compound A Compound B

Compound C

H

HH

H H

H

H

Cl

76

GO ON TO THE NEXT PAGE.GO ON TO THE NEXT PAGE.GO ON TO THE NEXT PAGE.

Full-Length Test 5 06/27/2003 12:18 PM Page 76

Page 77: MCAT Full Length5

207. Which of the following conformers of compound Ais lowest in energy?

208. What is hybridization of carbon I in compound C?

A. sp3

B. sp2

C. sp

D. There is no hybridization between the s and porbitals.

209. What would be the effect on Reaction 1 if CompoundC was reacted with HBr in the presence of UV light?

A. Compound C would undergo free-radical poly-merization.

B. An equal mixture of Compound C and Com-pound D would be formed.

C. Compound C would not react with HBr.

D. There would be no effect on the reaction.

Cl

ClCH3

CH3Br

Br

A.

ClCl CH3

CH3Br

Br

B.

Cl

Cl

CH3

CH3 BrBr

C.

Cl

Cl

CH3

CH3

Br

Br

D.

77

GO ON TO THE NEXT PAGE.GO ON TO THE NEXT PAGE.GO ON TO THE NEXT PAGE.

Full-Length Test 5 06/27/2003 12:18 PM Page 77

Page 78: MCAT Full Length5

Questions 210 through 214 are NOT basedon a descriptive passage.

210. When a person inhales fresh air:

A. oxygen diffuses from the alveoli into the pul-monary blood.

B. oxygen diffuses from the pulmonary blood intothe alveoli.

C. oxygen dissociates from hemoglobin in the pul-monary blood.

D. oxygen dissociates from hemoglobin in thealveoli.

211. Which of the following is NOT a function of bile?

A. Emulsification of fat particles into minute parti-cles that are susceptible to the actions of lipases

B. Neutralization of acid emptied into the largeintestine from the stomach

C. Transport of the end-products of fat digestionthrough the intestinal mucosal membrane

D. Excretion of bilirubin, the end-product ofhemoglobin destruction

212. How many peaks would be observed in the 1H NMRspectrum of C(CH3)4?

A. 1

B. 2

C. 4

D. 12

213. Parathyroid hormone, which is an antagonist of cal-citonin, stimulates activity in:

A. osteoclasts, which dissolve bone.

B. osteoblasts, which form bone.

C. chondrocytes, which secrete cartilage.

D. the kidneys, which excrete vitamin D.

214. Aldosterone is the adrenal hormone responsible forthe reabsorption of sodium ions from the distal tubuleand collecting duct and the secretion of potassiumions into them. Which of the following symptomswould be present in a patient with Addison's disease,which is the result of adrenocortical deficiency?

A. High blood pressure and low [K+] in the extra-cellular fluid.

B. Low blood pressure and low [K+] in the extra-cellular fluid.

C. Low blood pressure and high [K+] in the extra-cellular fluid.

D. No change in either blood pressure or [K+] inthe extracellular fluid.

STOP. IF YOU FINISH BEFORE TIME IS CALLED,CHECK YOUR WORK.YOU MAY GO BACK TO ANYQUESTION IN THIS SECTION ONLY.

78

Full-Length Test 5 06/27/2003 12:18 PM Page 78

Page 79: MCAT Full Length5

79

Verbal Reasoning Passages Copyright Permissions

From “The Deep Roots of Our Decline,” by John Attarian. The Wall Street Journal. Copyright © 1994.

From Duke Ellington, by James Lincoln Collier. Copyright © 1987.

From The Illusion of Peace: International Relations in Europe, 1918–1933, by Sally Marks. Copyright © 1976.

From Sex, Art and American Culture, by Camille Paglia. Copyright © 1992.

From Citizen Welles, by Frank Brady. Copyright © 1989.

From The Dreaming Brain, by J. Allan Hobson. Copyright © 1989 by Basic Books, Inc.

From Say the Right Thing—Or Else; Attack Ideas Not People, by Judith Martin and Gunther Stent. Copyright © 1991 by TheNew York Times Company.

From Strategies of Containment, by John Lewis Gaddis. Copyright © 1982 by Oxford University Press.

From The Human Evolution Coloring Book, by Adrienne L. Zihlman. Copyright © 1982 by Coloring Concepts, Inc.

Full-Length Test 5 06/27/2003 12:18 PM Page 79

Page 80: MCAT Full Length5

Full-Length Test 5 06/27/2003 12:18 PM Page 80

Page 81: MCAT Full Length5

Full-Length Test 5 06/27/2003 12:18 PM Page 81